Sunteți pe pagina 1din 1281
Ene) THOMAS / FINNEY 9 ue ELT TE ae A 7) P= ad or) = al — n Qu EDITION Calculus and Analytic Geometry George B. Thomas, Jr. Massachusetts Institute of Technology Ross L. Finney With the collaboration of Maurice D. Weir Naval Postgraduate School Addison-Wesley Publishing Company Reading, Massachusetts + Menlo Park, Clifomia + New York Don Mills, Ontario * Wokingham, England * Amsterdam Bonn + Sydney + Singapore + Tokyo + Madrid San Jun + Milan + Paris Acquisitions Editor Development Editor ‘Managing Editor Senior Produetion Supervisor Senior Marketing Manager Marketing Coordinator Prepress Buying Manager Art Buyer Senior Manufacturing Manager Manufacturing Coordinator World Student Series Laurie Rosatone __-Produetion Editorial Services Barbara Pendergast ‘Marianne Lepp Art Editors Susan London-Payne, Connie Hulse Karen Guardino Copy Editor Barbara Flanagan Jennifer Bagdigian Proofreader Joyce Grandy Andrew Fisher Text Design Martha Podren, Podren Design; Benjamin Rivera Geri Davis, Quadtrata, Ine. Sarah MeCracken Cover Design Marshall Henrich Joseph Vetere Cover Photo John Lund/Tony Stone Worldwide Roy Logan Composition TSI Graphies, Inc. Evelyn Beaton Technical Illustration Tech Graphies Photo Credits: 142, 238, 408, 633, 722, 875, 899, From PSSC Physics 2, 1965; D.C. Heath & Co. with Education Development Center, Inc., Newton, MA. Reprinted with permission 186, AP/Wide World Photos 266, Scott A. Burns, Urbana, IL 287, Joshua E. Barnes, Univer- sity of Hawaii 354, Marshall Henrichs 398, © Richard F. Voss/IBM Research 442, © Susan ‘Van Etten 872, AP/Wide World Photos 889, © 1994 Nelson L. Max, University of Califor- niaBiological Photo Service; Graphic by Alfred Gray 938, ND Roger-Viollet 1068, INASA/Jet Propulsion Reprinted with corrections, June, 1998, ‘Copyright © 1996 by Addison-Wesley Publishing Company, Inc. Al rights reserved. No part of this publication may be reproduced, stored ina retrieval system, or transmitted, inany form or by any means, electronic, mechanical, photocopying, or otherwise, without the prior written permission ofthe publisher. Printed in the United States of America, 456789 10-VH-99 98 ISBN 0-201-40015-4 Contents To the Instructor vill To the Student xi) Preliminaries Real Numbers and the Real Line 1 Coordinates, Lines, and Increments 8 Functions "17 Shifting Graphs 27 ‘Trigonometsic Funetions 35 QUESTIONS TO GUIDE YOUR REVIEW 47__—_-PRACTICEEXERCISES 48 ADDITIONAL EXERCISES—THEORY, EXAMPLES, APPLICATIONS. 49 Limits and 4.4 Rates of Change and Limits 51 nuit 112 — Rules for Finding Limits 61 ‘ontinui a Continuity 1.3. Target Values and Formal Definitions of Limits 66 14 Extensions of the Limit Concept 78 15 Continuity 87 166 Tangent Lines 97 QUESTIONS TO GUIDE Your REVIEW 103 PRACTICE EXERCISES 104 [ADDITIONAL EXERCISES—THEORY, EXAMPLES, APPLICATIONS 105 Derivatives 2.1 The Derivative of a Function 109 2.2 Differentiation Rules 121 2.3 Rates of Change 131 2.4 Derivatives of Tigonomeirie Functions 143 2.5 TheChain Rule 154 2.6 Implicit Differentiation and Rational Exponents 164 2.7 Related Rates of Change 172 ‘QUESTIONS To GUIDE YOUR REVIEW 180 PRACTICE EXERCISES 181 ADDITIONAL EXERCISES—THEORY, EXAMPLES, APPLICATIONS 185 Applications of 3.1 Extreme Values of Functions 189, Derivatives 3.2. TheMean Value Theorem — 196 3.3 TheFirst Derivative Test for Local Extreme Values 205 iv Contents Integration Applications of Integrals Transcendental Functions Techniques of Integration 34 36 37 38 5a 52 53 54 55 56 58 59 5.10 7A 72 Graphing with y and y” 209 Limits as.x £5, Asymptotes and Dominant Terms 220 Optimization 233 Linearization and Differentials 248 Newton's Method 260 QUESTIONS TOGUIDE YOUR REVIEW 268 PRACTICE EXERCISES 269, ADDITIONAL EXERCISES—THEORY, EXAMPLES, APPLICATIONS. 272 Indefinite Integrals 275 Differential Equations, Initial Value Problems, and Mathematical Modeling 282 Integration by Substitution—Running the Chain Rule Backward 290 Estimating with Finite Sums 298 Riemann Sums and Definite Integrals 309 Properties, Area, and the Mean Value Theorem — 323 ‘The Fundamental Theorem — 332 Substitution in Definite Integrals 342 Numerical Integration 346 QUESTIONS TOGUIDE YOUR REVIEW 356 PRACTICE EXERCISES 357 ADDITIONAL EXERCISES—THEORY, EXAMPLES, APPLICATIONS. 360) Areas Between Curves 365 Finding Volumes by Slicing 374 Volumes of Solids of Revolution—Disks and Washers 379 Cylindrical Shells 387 Lengths of Plane Curves 393 ‘Areas of Surfaces of Revolution 400 Moments and Centers of Mass 407 Work 418 Fluid Pressures and Forces 427 ‘The Basie Patern and Other Modeling Applications 434 (QUESTIONS TO GUIDE YOUR REVIEW 444 PRACTICEEXERCISES 444 ADDITIONAL EXERCISES—THEORY, EXAMPLES, APPLICATIONS. 447 Inverse Functions and Theit Derivatives 449 [Natural Logarithms 458 ‘The Exponential Function 467 a andlogsr 474 Growth and Decay 482 LHopitals Rule 491 Relative Rates of Growth 498 Inverse Trigonometric Functions S04 Derivatives of Inverse Trigonometric Functions; Hyperbolic Functions 520 First Onder Differential Equations 529 Euler's Numerical Method; Slope Fields 541 (QUESTIONS TO GUIDE YOUR REVIEW S47 PRACTICE EXERCISES 548, ADDITIONAL EXERCISES —THEORY, EXAMPLES, APPLICATIONS. SSI ltegrals S13 Basi Integration Formulas 555 Integration by Parts 562 Infinite Series Conic Sections, Parametrized Curves, and Polar Coordinates Vectors and Analytic Geometry in Space Vector-Valued Functions and Motion in Space 73 75 76 104 102 103 104 105 106 107 Wt 12 13 4 U5 Contents Partial Fractions 569 ‘Trigonometric Substitutions 578 Integral Tables and CAS 583 Improper Integrals 594 QUESTIONS TO GUIDE YOUR REVIEW 606 PRACTICE EXERCISES 606 ADDITIONAL EXERCISES—THEORY, EXAMPLES, APPLICATIONS. 609) Limits of Sequences of Numbers 613 ‘Theorems for Calculating Limits of Sequences 622 Infinite Series 630 ‘The Integral Test for Series of Nonnegative Terms 640 ‘Comparison Tests for Series of Nonnegative Terms 644 The Ratio and Root Tess for Series of Nonnegative Terms 649 ‘Altemating Series, Absolute and Conditional Convergence 655, Power Series 663 ‘Taylorand Maclaurin Series 672 Convergence of Taylor Series; Exror Estimates 678 Applications of Power Series 688 QUESTIONS TO GUIDE YouR REVIEW 699_-PRACTICE EXERCISES 700 ADDITIONAL EXERCISES—THEORY, EXAMPLES, APPLICATIONS 703 Conic Sections and Quadratic Equations 709 Classifying Conie Sections by Eccentricity 723, (Quadratic Equations and Rotations 728 Parametrzations of Plane Curves 734 CCaleulus with Parametrized Curves 744 Polar Coordinates 751 Graphing in Polar Coordinates 756 Polar Equations for Conie Sections 764 Integration in Polar Coordinates 770 (QUESTIONS TO GUIDE YouR REVIEW 777 PRACTICE EXERCISES 778 ADDITIONAL EXERCISES—THEORY, EXAMPLES, APPLICATIONS 783 ‘Vectors inthe Plane 787 Cartesian (Rectangular) Coordinates and Vectors in Space 795 Dot Products 806 Cross Products 815 Lines and Planes in Space 822 Cylinders and Quadric Surfaces 829 Cylindrical and Spherical Coordinates 841 Questions To GUIDE Your REVIEW $47___-PRACTICE EXERCISES 848 ADDITIONAL EXERCISES—THEORY, EXAMPLES, APPLICATIONS 851 Vector-Valued Functions and Space Curves 855 ‘Modeling Projectile Motion 868 ‘Are Length and the Unit Tangent Vector T 876 Curvature, Torsion, and the TNB Frame $81 Planetary Motion and Satellites 893 QUESTIONS TOGUIDE YOUR REVIEW 902___-PRACTICE EXERCISES 902 ADDITIONAL EXERCISES THEORY, EXAMPLES, APPLICATIONS 905 v vi Contents Multivariable Functions and Partial Derivatives Multiple Integrals Integration in Vector Fields Appendices 124 122 123 12 2s 126 127 128 129 12.10 134 132 133 134 135 136 137 Answers Index Functions of Several Variables 909 Limits and Continuity 917 arial Derivatives 924 Differeniabilty, Linearization, and Differeatials 933, ‘The Chain Rule 944 Partial Derivatives with Constrained Variables 952 Directional Derivatives, Gradient Vectors, and Tangent Planes 957 [Extreme Values and Saddle Points 970 Lagrange Multipliers 980 ‘Taylor's Formula 989 QUESTIONS TO GUIDE YOUR REVIEW 993_PRACTICE EXERCISES 994 ADDITIONAL EXERCISES—THEORY, EXAMPLES, APPLICATIONS 998 Double Integrals 1001 ‘Areas, Moments, and Centers of Mass 1012 Double Integrals in Polar Form 1020 Triple integrals in Rectangular Coordinates 1026 ‘Masses and Moments in Three Dimensions 1034 ‘Triple Integrals in Cylindrical and Spherical Coordinates 1039 Substitutions in Multiple Integrals 1048 QUESTIONS TOGUIDE YOURREVIEW 1055 PRACTICE EXERCISES 1056 ADDITIONAL EXERCISES—THEORY, EXAMPLES, APPLICATIONS. 1058 Line Integrals 1061 Vector Fields, Work, Circulation, and Flux 1067 Path Independence, Potential Functions, and Conservative Fields 1076 Green's Theorem in the Plane 1084 Surface Area and Surface Integrals 1096 Paramettized Surfaces 1106 Stokes's Theorem 1114 The Divergence Theorem and a Unified Theory 1123 (QUESTIONS TO GUIDE YOUR REVIEW 134 PRACTICE EXERCISES 1134 ADDITIONAL EXERCISES—THEORY, EXAMPLES, APPLICATIONS 1137 Mathematical Induction —A-1 roofs of Limit Theorems in Section 1.2. A4 Complex Numbers A-7 ‘Simpson's One-Third Rule A-I7 CCauchy’s Mean Value Theorem end the Stronger Form of Hopital’s Rule A-18 Limits That Arise Frequently A-20 ‘The Distributive Law for Vector Cross Products A-21 Determinants and Cramer's Rule A-22 Euler's Theorem and the Increment Theorem A-29 35 M A Brief Table of Integrals 1 CAS Explorations and Projects (Listed by chapter and section) Preliminaries P4 How the graph of y = flax is affected by changing «| P5 How the graph of fx)=A sin (2R/B\x— CO) +D responds to changes in A,B, and D Chapter Limits and Continuity 1.1 Comparing graphical estimates of limits with CAS symbolic limit calculations 1.3__Exploring the formal definition of limit by finding deltas for specitcepsitons graphically 1.6 Observing the convergence of secant lines to tangent lines Chapter2 Derivatives 2.1 Given (x, find x) as limit. Compare the graphs off and and plo selected tangents 2.6 Differentiate implicitly and plot implicit curves together with tangent ines Chapter 3. Applications of Derivatives 3.1 Finding absolute extrema by analyzing f and f” ‘numerically and graphically 3.7 Estimating the eror ina linearization by ploting f(x) Us), and [f(8) = LO] Chapter 4 _ Integration 44 Find the average value off (1) and the point or points, ‘where itis assumed 45. Exploring Riemann sums and thei sits 47 1) Investigating the relationship of F(x) = [20 dt Foryand 0) by Analyzing Fox) = [24nd Chapter 5 Application of integrals 5.1 Finding intersections of eurves 5.5 Are length estimates Chapter 6 Transcendental Functions 6.1 Graphing inverse functions and their derivatives 6.12 Exploring differential equations graphically and numeri- cally with slope fields and Euler approximations Chapter 7 Techniques of Integration 75 Usinga CAS to integrate. An example of a CAS-resistant incegral 7.6 Exploring the convergence of improper integrals Chapter 8 Infinite Series 8.1 Exploring the convergence of sequences, Compound intrest with deposits and withdrawals. The logistic diference equation and chaotic behavior 85 Exploring E7. (tMmn'sin?n), a series whose conver- gence or divergence has not yet been determined 8.10 Comparing functions’ linear quadratic, and cubic approximations Chapter 9 Conic Sections, Parametrized Curves, and Polar Coordinates 9.5 Exploring the geometry of curves that are defined implic: itly or explicitly by parametric equations. Numerical estimates ofthe lengths of nonelementary paths 9.8 How the graph of r= ke] + ¢ cos @) isalfected by changes ineand &. How the ellipse r= a(1 ~e"/(1 +e cos 8) responds to changes ina and e Chapter 10 Vectors and Analytic Geometry in Space 10.6 Viewing quadrc surfaces ftom different postions 10.7 Equations of spheres in cylindrical, spherical, and rectan- gular coordinate systems: Coordinate conversions and surface plots Chapter 11. Vector-Valued Functions and Motion in Space 11.1. Ploting tangents to space curves. Exploring the general helix 11.4 Finding and ploting circles of curvature in the plane. Finding x, 7, T, N, and B for curves in space Chapter 12 Multivariable Functions and Partial Derivatives 12.1 Ploiting surfaces z= f(x,y) and associated level curves. Implicit and parametrized surfaces 12.8 Classifying critical points and identifying extreme values ‘sing information gathered from surface plots, level curves, and discriminant values 12.9 Implementing the method of Lagrange multipliers for functions of tare and four independent variables Chapter 13 Multiple Integrals 13.3 Changing Cartesian integrals into equivalent polar itograls for evaluation 13.4 Evaluating tiple integrals over solid regions Chapter 14 Integration in Vector Fields 14.1" Evaluating Jo's » 2) ds numerically 142. Estimating the work done by a vector field along a given path in space 144 Applying Green's theorem to find counterclockwise circulation vi viii To the Instructor This Is a Major Revision ‘Throughout the 40 years that it has been in print, Thomas/Finney has been used to suppor a variety of teaching methods from traditional to experimental. In response to the many exciting currents in teaching calculus in the 1990s, the new edition is the most extensive revision of Thomas/Finney ever, We have built on the traditional strengths of the book—excellent exercises, sound mathematics, variety in applica- tions—to produce a flexible text that contains all the elements needed to teach the many different kinds of courses that exist today. ‘A book does not make a course: The instructor and the students do. With this in mind we have added features to Thomas/Finney 9th edition ro make it the most flex ible calculus teaching resource yet. + The exercises have been reorganized to facilitate assigning a subset of the ‘material ina section. + The grapher explorations, all accessible with any graphing calculator, many suitable for in-class and group work, have been expanded. + New Computer Algebra System (CAS) explorations and projects that re- quite a CAS have been included. Some of these can be done quickly while others require several hours. All are suitable for either individual or group ‘work. You will find a list of CAS exercise topics following the Table of Contents * ‘Technology Connection notes appear throughout the text suggesting experi- ments students might do with a grapher to supplement their understanding of a given topic. These notes are meant to encourage students to think of their grapher ass casually available 100}, like a pencil * We revised the entre first semester and large parts of the second and third semesters to provide what we believe is cleaner, more visual, and more ac- cessible book. With all these changes, we have not compromised our belief that the fundamental goal of a calculus book is to prepare students to enter the scientific community. Students Will Find Even More Support for Creative Problem Solving Throughout this book, we have included examples and discussions that encourage students to think visually and numerically. Almost every exercise set has easy to Tothe Instructor ix Technology Tage Values You cam exponent with ages values om 8 graphing uly. Graph the function together with a target imteral defined by horizontal lines above and teow the propose limit. Adjust the ange ers zoom unl the fuscon’sHehuor inside the target itera ela Then ‘serve whl appers shen you try to nd a infra of vale tha wil ep the fein vas within the target interval (Se alo Exercises Td Teer and CAS Eseries 61-64), For example, ry this or f(s) = FZ andthe tng interval (18,22) Keeping x between 1.75 ana 2.28 wil ‘on the yrais Tha s grph jy 4) andthe lines 9p = 1.8 95 = 22. Then Keep between 18 ond 22 ty the tet itera (198, 2.02) and (9898, 20002 mid-level exercises that require students to generate and interpret graphs eee nenen as a tool for understanding mathematical or real-world relationships. eee ee ee setacumsone | Many sections also contain a few more challenging problems to extend be ve nl 231 the range of the mathematically curious. 1 Toe gph of Fr made of ced Iie eae nd This edition has more than 2300 figures to appeal to the students” era eee ame ae geometric intuition. Drawing lessons aid students with difficult 3- Ai) Te drive of ee se oc a Fe. 2.18 ‘dimensional sketches, enhancing their ability to think in 3-space. In this ‘edition we have increased the use of visualization internal to the discus- sion. ‘The burden of exposition is shared by art in the body of the text when we feel that pictures and text together will convey ideas better than words alone. ‘Throughout the text, students are asked to experiment, investigate, and explain, Writing exercises are placed throughout the text, In addi- tion, each chapter end contains alist of questions that ask students to re 2.19 The dervatve aah for Exerc 22 view and summarize what they have leamed. Many of these exercises make good writing assignments. Rees pt a sing tat te pp sans a (-2,0) iste O(-2.3, WS, stot are sec seep monseet Comune? os 149 xodg mys about tangent ne Students Will Master Techniques Problem Solving Strategies We believe thatthe students learn best when proce- dural techniques are Iaid out as clearly as possible. To this end we have revisited the summaries of the steps used to solve problems, adding some where necessary, delet- ing some where a thought process rather than a technique was at issue, and making each one clear and useful. As always, we are especially careful that examples in the text follow the steps outlined by the discussion Exercises Every exercise set has been reviewed and revised. Exercises are now grouped by topic, with special sections for grapher explorations. Many sections also x To the Instructor Hidden Behavior Sometimes eetine bp otf fore have a set of Computer Algebra System (CAS) Explorations and Projets, anew fea- ture for this eition. Within each group, the exercises are graded and paired. Within this framework, the exercises generally follow the order of presentation of the text Exercises that require a calculator or computer are identified by icons: calcu- Iator exercise, 8 graphing utility (suchas graphing calculator) exercise, and @Com- puter Algebra System exercise Checklist for Graphing a Function y 4. Look for symmeuy. Sof il ge ‘ue rg pce 2. Ie de fuetion a sito «known fanton? ‘3 Analyze dominant erm. Divide ation arcuors to polynomial + remainde 4. Check for asymptote and removable discontinuities. 1s there aero denominator a any point? ‘What happens as > 00? Compute fond solve f° 0, Kenly criss pons snd determine intra of ie and fall. Compt "to determine conesvty and infston points. ‘Sketch the griph's peal shape volute spacial vaes (endpoints, cial points, iatereeps) CGeaph fusing dominant tems, general shape. and special pints for Within the exercise sets, we have practice exercises, exervises that encourage ctitical thinking, more challenging exercises (in subsections marked “Applications and Theory"), and exercises that require writing in English about concepts, Writing exercises are placed both throughout the exercise sets, and in an end-of-chapter fea- ture called “Questions to Guide Your Review.” Chapter End At the end of each chapter are three features with questions that summarize the chapter in different ways. Questions to Guide Your Review ask students to think about concepts and ver~ balize their understanding without trying to calculate numeric answers. These are, as always, suitable for writing exercises. Practice Exercises provide a review of the techniques, ideas, and key applica~ tions. Additional Exercises—Theory, Examples, Applications supply challenging ap- plications and theoretic problems that deepen the understanding of mathemati- cal ideas, Applications, Technology, History—Features That Bring Calculus to Life Applications and Examples _ It has been a hallmark of this book through the years that we illustrate applications of calculus with eal data based on already familiar sit tations or situations students are likely to encounter soon, Throughout the ext, we cite sources forthe data and/or articles from which the applications are drawn, help ing students understand that calculus is a current, dynamic field, Most ofthese appli- iar, cxibootsaapiscement To End te volume of water hc! = slo the cmon pace ofan ie ‘tite lo 10 soir of oul gh, ease ess Seaton wen As) ofthe abe pron af eal th fre poi anh ne Sion eo xine he ae (Baot A(x) fom one end fthe nae w he ede The able eras te mes ences “Sten” 0 tag 10a {he prton pons we cae fr th eatng gp Ppt, ‘shor here. The conston sisi eg (stance betes mca ons) = 256 8 abst? 6 1/2, ten or ‘ae comenience of he bier. \\ —\\ 2) Enime Pipa’ piconet volume to he meet 1D) The gues inthe tale ae for sant, which weighs | Su aw iy sd of at is Pra | ‘te? Digtncenen een ponds or sal cra, ed 4 Tong tne eng ton = 220 or ager vse) Tothe Instructor xi cations are directed toward science and engineering, but there are ‘many from biology and the social sciences as well. ‘Technology: Graphing Calculator and Computer Algebra Sys- tems Explorations Virtually every section of the text contains cal culator exercises that explore numerical pattems and/or graphing calculator exercises that ask students to generate and interpret graphs as a tool to understanding mathematical and real-world rela- tionships. Many of the calculator and graphing calculator exercises are Suitable for classroom demonstration or for group work by stu- dents in or out of class. ‘Computer Algebra System (CAS) exercises have been added 10 every chapter. These exercises, 160 in all, have been tested on both ‘Mathematica and Maple. A full list of CAS exercise topics follows the Table of Contents, As in previous editions, soc" has been 80 that its range,(0, 1/2) U (W/2, w], and derivative, 1/(iel Ve ~ 1), agree with the results returned by Computer Algebra Systems and scientific cal- culators. ‘Notes appear throughout the text encouraging students to ex plore with graphers. History Any student is enriched by seeing the human side of mathematics. As in earlier editions, we feature history boxes that de- scribe the origins of ideas, conflicts concerning ownership of ideas, and interesting sidelights into popular topics such as fractals and chaos, The Many Faces of This Book Mathematics Is a Formal and Beautiful Language A good part of the beauty of the calculus les inthe fact that it isa stunning cre- ation ofthe human mind. As in previous editions we have been care- ful to say only what is true and mathematically sound. In this edition we reviewed every definition, theorem, corollary, and proof for clarity and mathematical conrect- ness, Even Better Suited to Be the Reference Text in a Reform Course Whether cal- clus is taught by a traditional lecture or entirely in labs with individual and group learning which focuses on numeric and graphical experimentation, ideas and tech- niques need to be articulated clearly. This book provides the exercises for computer ‘and grapher experiments and group learning and, ina traditional format, the summa- tion of the lesson—the formal statement of the mathematics and the clear presenta- tion of the technique. ‘Students Will Learn from This Book for Many Years to Come We provide far ‘more material than any one instructor would want to teach. We do this intentionally. Students can continue to learn calculus from this book long after the class has ended. It provides an accessible review of the calculus a student has already studied. It is a resource for the working engineer or scientist, xii To the Instructor Content Features of the Ninth Edition Tothe Instructor xi Supplements for the Instructor OmniTest? in DOS-Based Format: This easy-to-use software is developed ex- clusively for Addison-Wesley by ips Publishing, a leader in computerized testing and assessment. Among its features are the following. ‘+ DOS interface is easy to learn and operate. The windows look-alike inter face makes it easy to choose and control the items as well as the format for each test. * You can easily create make-up exams, customized homework assign- ments, and multiple test forms to prevent plagiarism. OmniTest’ xiv To the Instructor algorithm driven—meaning the program can automatically insert new num- bers into the same equation—creating hundreds of variations of that equa- tion. The numbers are constrained to keep answers reasonable. This allows you to create a virtually endless supply of parallel versions of the same test. ‘This new version of OmniTest also allows you to “lock in” the values shown in the model problem, if you wish. ‘© Test items are keyed by section to the text. Within the section, you can se~ lect questions that test individual objectives from that section. ‘© You can enter your own questions by way of OmniTest™’s sophisticated editor—complete with mathematical notation. Instructor's Solutions Manual by Maurice D. Weir (Naval Postgraduate School), ‘This two-volume supplement contains the worked-out solutions for all the exercises in the text, Answer Book contains short answers to most exercises in the text. Supplements for the Instructor and Student Student Study Guide by Maurice D. Weir (Naval Postgraduate School). Orga- nized to correspond with the text, this workbook in a semiprogrammed format in- creases student proficiency with study tips and additional practice. Student Solutions Manual by Maurice D. Weir (Naval Postgraduate School). ‘This manual is designed for the student and contains carcfully worked-out solutions to all of the odd-numbered exercises in the text Differential Equations Primer A short, supplementary manual containing ap- proximately a chapter's-worth of material, Available should the instructor choose to cover this material within the calculus sequence. Technology-Related Supplements Analyzer’ This program is a tool for exploring functions in calculus and many ‘other disciplines. It can graph a function of a single variable and overlay graphs of other functions. It can differentiate, integrate, or iterate a function. It can find roots, maxima and minima, and inflection points, as well as vertical asymptotes. In addi- tion, Analyzes" can compose functions, graph polar and parametric equations, make “nitines Br eat vres antrnab raimated.-seyances. with changing.narameters. It eX- piloits the unique flexibility of the Macintosh wherever possible, allowing input to be either numeric (from the keyboard) or graphic (with a mouse). Analyzer* runs on Macintosh II, Plus, or better. ‘The Calculus Explorer Consisting of 27 programs ranging from functions to vec {or fields, this software enables the instructor and student to use the computer as an “electronic chalkboard.” The Explorer is highly interactive and allows for manipula- tion of variables and equations to provide graphical visualization of mathematical relationships that are not intuitively obvious. The Explorer provides user-friendly operation through an easy-to-use menu-criven system, extensive on-line documenta- tion, superior graphics capability, and fast operation. An accompanying manual in- Tothe Instructor xv cludes sections covering each program, with appropriate examples and exercises. Available for IBM PC/compatibles. InSight A calculus demonstration software program that enhances understanding ‘of calculus concepts graphically. The program consists often simulations. Each pre- sents an application and takes the user through the solution visually. The format is interactive. Available for IBM PC/compatibles. Laboratories for Calculus 1 Using Mathematica By Margaret Hoft, The Univer- sity of Michigan—Dearbomn. An inexpensive collection of Mathematica lab experi- ments consisting of material usually covered in the first term of the calculus se- quence. Math Explorations Series Each manval provides problems and explorations in calculus. Intended for self-paced and laboratory settings, these books are an excel- lent complement tothe tex. Exploring Calculus with a Graphing Calculator, Second Edition, by Char- lene E, Beckmann and Ted Sundstrom of Grand Valley State Universit: Exploring Caleulus with Mathematica, by James K. Finch and Millianne Lehmann of the University of San Francisco, Exploring Calculus with Derive, by David C. Amey of the United States Mili tary Academy at West Point Exploring Caleulus with Maple, by Mark H. Holmes, Joseph G. Ecker, William E. Boyce, and William L. Seigmann of Rensselaer Polytechnic Insti- tute. Exploring Caleulus with Analyzer*, by Richard E. Sours of Wilkes Univer- sity. Exploring Calculus with the IBM PC Version 20, by John B. Fraleigh and Lewis I Pakula of the University of Rhode Island, xvi To the Instructor Acknowledgments ‘We would like to express our thanks forthe many valuable conti- butions of the people who reviewed this book as it developed though its various stages: Manuscript Reviewers Erol Barbut, University of Idaho Neil E. Berger, University of Mlinois at Chicago George Bradley, Duquesne University ‘Thomas R. Caplinger, Memphis Siae University Counts L. Card, Black Hills Stare University James C. Chesla, Grand Rapids Community College PML. Dearing, Clemson University Maureen H. Fenrick, Mankato State University Stuart Goldenberg, CA Polytechnic State University Johnny L. Henderson, Auburn University James V. Herod, Georgia Institue of Technology Paul Hess, Grand Rapids Community College Alice J. Kelly, Santa Clara University ‘evel G. LaTorre, Clemson University Pamela Lowry, Lawrence Technological University John E. Marin, Il, Santa Rosa Junior College James Martino, Johns Hopkins University James R. McKinney, California State Polytechnic University Jelf Morgan, Texas A & M University F.J. Papp, University of Michigan—Dearbor Peter Ross, Santa Clara University Rouben Rostamian, University of Maryland Baltimore County William L. Siegmann, Rensselaer Polytechnic Institute John R. Smart, University of Wisconsin—Madison Dennis C. Smolarski, S.J, Santa Clara University Bobby N. Winters, Pnsburgh State University ‘Technology Notes Reviewers Lynn Kamstra Ipina, University of Wyoming Robert Flagg, University of Southern Maine Jelfiey Stephen Fox, University of Colorado at Boulder James Martino, Johns Hopkins University Carl W. Mortis, University of Missouri—Columbia Robert G. Stein, Califia State University—San Bernardino Accuracy Checkers Steven R. Finch, Massachusetts Bay Community College Paul R. Loreask, MathSoyt, Inc. John R. Martin, Tarrant County Junior College Jeffrey D. Oldham, Stanford University Exercises In addition, we thank the following people who reviewed the exer- cise sets for content and balance and contributed many of the in teresting new exercises: Jennifer Earles Szydlik, University of Wisconsin—Madison Aparna W. Higgins, University of Dayton William Higgins, Wittenberg University ‘Leonard F. Klosinski, Santa Clara University David Mann, Naval Postgraduate School Kirby C. Smith, Texas A & M University Kirby Smith was also a pre-revision reviewer and we wish to thank him for his many helpful suggestions. ‘We would like to express our appreciation to David Cantight, Naval Postgraduate School, for his advice and his contributions to the CAS exercise sets, and Gladwin Bartel, at Otero Junior Col- lege, for his many helpful suggestions Answers ‘We would lke to thank Cynthia Hutcherson for providing answers for exercises in some of the chapters inthis edition. We also ap- preciate the work of an outstanding team of graduate students at Stanford University, who checked every answer in the text for ac- ‘curacy: Miguel Abreu, David Cardon, Tanya Kalich, Jeffrey D. Oldham, and Julie Roskies. Jeffrey D. Oldham also tested all the CCAS exercises, and we thank him for his many helpful sugges- Other Contributors We are particularly grateful to Maurice D. Weir, Naval Postgrad. ate Schoel, who shared his teaching ides throughout the prepara tion ofthis book. He produced the final exercise sets andl wrate ‘most of the CAS exercises for this edition, We appreciate his con- stant encouragement and thoughtful advice. We thank Richard A. Askey, University of Wisconsin—Madi- soa, David MeKay, Oregon State University, and Richard G. Montgomery, Southern Oregon State College, for sharing their teaching ideas for this edition. We are also grateful to Erich Laurence Havenstein, College of DuPage, for generously providing an improved treatment of ‘chaos in Newton's method, and to Robert Carlson, University of Colorado, Colorado Springs, for improving the exposition inthe section on relative rates of growth of functions. To the Student What Is Calculus? Caleutus is the mathematics of motion and change. Where there is motion or growth, where variable forces are at work producing acceleration, calculus isthe right math- ematics to apply. This was true in the beginnings of the subject, and itis true today. Calculus was first invented to meet the mathematical needs of the scientists of the sixteenth and seventeenth centuries, needs that were mainly mechanical in na- ture. Differential calculus dealt with the problem of calculating rates of change. It enabled people to define slopes of curves, to calculate velocities and accelerations of moving bodies, to find firing angles that would give cannons their greatest range, and to predict the times when planets would be closest together or farthest apart. In- tegral calculus dealt with the problem of determining a function from information about its rate of change. It enabled people to calculate the future location of a body from its present position and a knowledge of the forces acting on it, to find the areas of irregular regions in the plane, to measure the lengths of curves, and to find the volumes and masses of arbitrary solid. ‘Today, calculus and its extensions in mathematical analysis are far reaching in- deed, and the physicists, mathematicians, and astronomers who first invented the subject would surely be amazed and delighted, as we hope you will be, to see what a profusion of problems it solves and what a range of fields now use it in the mathe- ‘matical models that bring understanding about the universe and the world around us. ‘The goal of this edition is to present a modern view of calculus enhanced by the use of technology. How to Learn Calculus Learning calculus is not the same as learning arithmetic, algebra, and geometry. In those subjects, you learn primarily how to calculate with numbers, how to simplify algebraic expressions and calculate with variables, and how to reason about point. lines, and figures in the plane. Calculus involves those techniques and skills but de- vetops others as well, with greater precision and at a deeper level. Calculus intro: duces so many new concepts and computational operations, in fact, that you will no longer be able to lear everything you need in class. You will have to learn a fair amount on your own or by working with other students. What should you do to learn? 1, Read the text. You will not be able to learn all the meanings and connections you need just by attempting the exercises. You will need to read relevant avi xvili To the Student passages in the book and work through examples step by step. Speed reading ‘will not work here, You are reading and searching for detail in a step-by-step logical fashion. This kind of reading, required by any deep and technical con- tent, takes attention, patience, and practice 2. Do the homework, keeping the following principles in mind. a) Sketch diagrams whenever possible. ) Write your solutions in a connected step-by-step logical fashion, as if you ‘were explaining to someone else. ©) Think about why each exercise is there. Why was it assigned? How is it re- lated (o the other assigned exercises? 3. Use your caleulator and computer whenever possible. Complete as many gra- ppher and CAS (Computer Algebra System) exercises as you can, even if they are rnot assigned. Graphs provide insight and visual representations of important concepts and relationships. Numbers can reveal important patterns. A CAS gives you the freedom to explore realistic problems and examples that involve calculations that are too difficult or lengthy to do by hand. 4. Try on your own to write short descriptions of the key points each time you complete a section of the text. If you succeed, you probably understand the ma- terial. IF you do not, you will know where there is a gap in your understanding. Learning calculus is a process—it does not come all at once. Be patient, perse~ vere, ask questions, discuss ideas and work with classmates, and seek help when you need it, right away. The rewards of learning calculus will be very satisfying, both in- tellectually and professionally. GBA, Ir, State College, PA RLF, Monterey, CA Preliminaries Overview This chapter reviews the main things you need to know to start calculus. The topics include the real number system, Cartesian coordinates in the plane, straight lines, parabolas, circles, functions, and trigonometry. Real Numbers and the Real Line This section reviews real numbers, inequalities, imtervals, and absolute values. Real Numbers and the Real Line Much of calculus is based on properties of the real number system. Real numbers are numbers that can be expressed as decimals, such as 3 073 4 = 7075000. 1 5 = 0.33333. 1.4142 ‘The dots ... in each case indicate that the sequence of decimal digits goes on forever. ‘The real numbers can be represented geometrically as points on a number line called the real line fi eh a 72 a aa 4 ‘ 3 ‘The symbol R. donotes either the real number system or, equivalently, the real line. Properties of Real Numbers ‘The properties of the eal number system fall nto three categories: algebraic prop- cnties, order properties, and completeness. The algebraic properties say that the real rumbers can be added, subtracted, multiplied, and divided (except by 0) to produce ‘more real numbers under the usual rules of arithmetic. You can never divide by 0. 1 2 Preliminaries ‘The order properties of real numbers are summarized in the following list. The symbol => means “implies. Notice the rules for multiplying an inequality by a number. Multiplying by a positive number preserves the inequality; multiplying by a negative number reverses the inequality. Also, reciprocation reverses the inequality for ‘numbers of the same sign. Rules for Inequalities Ifa, b, and c are real numbers, then: L acb=ate0=>ac —b <—a ns 1 1 6 Wa and b are both postive or both negative, then a < b= 5 < The completeness property of the real number system is deeper and harder to define precisely. Roughly speaking, it says that there are enough real numbers to “complete” the real number line, in the sense that there are no “holes” or “gaps” in it, Many of the theorems of calculus would fail ifthe real number system were not complete, and the nature of the connection is important, The topic is best saved for a more advanced course, however, and we will not pursue it Subsets of R We distinguish three special subsets of real numbers, 1. The natural numbers, namely 1,2, 3,4, 2. The integers, namely 0, +1, +2, +3, 3. The rational numbers, namely the numbers that can be expressed in the form of a fraction m,n, where m and m are integers and n # 0. Examples are 1 4 200 7 —p pp md STF 3 ‘The rational numbers are precisely the real numbers with decimal expansions that are either 8) terminating (ending in an infinite string of zeros), for example, Sc orso...2075 ) repeating (ending with a block of digits that repeats over and over), for example 2B The bar indicates the 23 _ 2.090909... = 2.09, block of repeating i sits The set of rational numbers has all the algebraic and order properties of the real numbers but lacks the completeness property. For example, there is no rational number whose square is 2; there is a “hole” in the rational line where 2 should be. Table 1. Types of intervals 1. Real Numbers and the Real Line 3 Real numbers that are not rational are called irrational numbers. They ae cher acterized by having nonterminating nd nonrepeating decimal expansions. Examples are n, V2, 95, and logi93 Intervals ‘A subset ofthe real line is called an interval if it contains atleast two numbers and Contains all the real numbers lying between any two of its elements. For example, the set of all real numbers x such that x > 6 is an interval, as isthe set of all x such that —2 < x <5. The set of all nonzero real numbers is not an interval; since 0 is absent, the set fails to contain every real number between —1 and 1 (for example). Geometrically, intervals correspond to rays and line segments on the real line, along with the real line itself. Intervals of numbers corresponding to line segments are finite intervals; intervals corresponding to rays and the real line are infinite intervals ‘A finite interval is said to be elosed if it contains both of its endpoints, half open if it contains one endpoint but not the other, and open if it contains neither endpoint. The endpoints are also called boundary points; they make up the in- {erval’s boundary. ‘The remaining points of the interval are interior points and together make up what is called the interval’s interior. Finite: Notation (6) (a,b) [a,d) (ab) (2,00) [a,00) co (20,21 (20,00) Set Graph trla a} tele 2a) fair numbers) 4 Preliminaries 1 Solutions for Example 1 Solving Inequalities ‘The process of finding the interval or intervals of numbers that satisfy an inequality in x is called solving the inequality EXAMPLE 1 Solve the following inequalities and graph their solution sets on the real line. a) 2 -1e5 can hold only if x > 1, because otherwise 6/(& — 1) is undefined or negative. Therefore, the inequality will be preserved if we multiply both sides by (x ~ 1), and we have rues 6251-5 satiny ban sion byte NESr Adds ite Bee oneet ‘The solution set is the half-open interval (1, 11/5] (Fig. 1c). a Absolute Value ‘The absolute value of a number x, denoted by |x|, is defined by the formula x x20 l= xy x<0. EXAMPLE2 — |3|=3, |0/=0, |-S|=~(-5)=5, |-lal[=l4l Notice that |x| > 0 for every real number x, and |x| = 0 if and only if x =0. It is important to remember that Ya = a}. Do not write Va? = a unless you already know that a > 0, ite tl= tal = ; aver t 4 2 Absolute values give distances between points on the number line Notice that absolute value bars in expressions like | ~3 +5] also work like parentheses: We do the arithmetic inside before taking the absolute value. 1. Real Numbers and the Real Line 5 Since the symbol J/@ always denotes the nonnegative square root of a, an alternate definition of |x| is [xl = v9". Geometrically, |x| represents the distance from x to the origin 0 on the real line. More generally (Fig. 2) lx — yl = the distance between x and y. ‘The absolute value has the following properties. Absolute Value Properties 1 |-a| =Ia\ ‘A number and its negative have the same absolute value, 2. ab| = |a\ibl The absolute value of a product is the product of the absolute values eel ‘The absolute value of a quotient isthe quotient of bh the absolute values. 4. |a-+6] <|al+ [6] The triangle inequality The absolute value of the sum of two numbers is less than or equal to the sum of their absolute values. Wa and b differ in sign, then |a + 6] is less than [a] + [b]. In all other cases, Ja + bj equals |al + |b EXAMPLE 3 [-3451= 221 <|-3+5) 3 +5] = 181 = 131+ 15] [-3-51=1- 3141-51 a EXAMPLE 4 Solve the equation [2x ~ 3) 1 Solution ‘The equation says that 2x — £7, so there are (wo possibilities M327 3-7 itch sae 2x = 10 2x = 4 Solve as su x= xa-2 The solutions of [2x — 3] = 7 are x = 5 and x = —2, Q Inequalities Involving Absolute Values The inequality |a| < D says that the distance from a to 0 is less than D. Therefore, ‘a must lie between D and —D. 6 Preliminaries ‘The symbol < means “if and only if," or “implies and is implied by.” ¢ 3 ry 3 The solution set of the inequality x~5) <9 isthe interval (—4, 14) graphed here (Example 5). 4 Graphs ofthe solution sets (a) [1,21 and (b) (--, 1]U 2, «) in Example 7 Intervals and Absolute Values If D is any positive number, then lal 2040) 1 7 6-x 3-4 9-5 ETE wo, SSF Be 4 L EHS 12430 Wh. 5x9) < 30-6) wm, 22. 2t Solve the inequalities in Exercises 19-34, expressing the solution sets as intervals of unions of intervals. Also, graph each solution set on the real line 19, ni <2 ma iris? a. W113 BtAs 3 |t4o1 x)? of -al>§ Quadratic Inequalities Solve the inequalities in Exercises 35-42. Express the solution sets 2 intervals of unions of intervals and graph them. Use the result Vat = [a as appropriate B=? 340 Md Lope! 2 Bj<< 7 BA —1/2, what can you say about x? la) |6| for any numbers a and b, 'b) Confirm your findings in (a) algebraically Coordinates, Lines, and Increments This section reviews coordinates and lines and discusses the notion of increment. Cartesian Coordinates in the Plane ‘The positions of all points in the plane can be measured with respect to two perpendicular real lines in the plane intersecting in the O-point of each (Fig. 5). ‘These lines are called coordinate axes in the plane. On the horizontal x-axis, numbers are denoted by x and increase to the right. On the vertical y-axis, numbers are denoted by y and increase upward. The point where x and y are both O is the origin of the coordinate system, often denoted by the letter O. If P is any point in the plane, we can draw lines through P perpendicular to the two coordinate axes. If the lines meet the x-axis at a and the y-axis at B, then a is the x-coordinate of P, and b is the y-coordinate. The ordered pair (a, b) is the Positive sais point’s coordinate pair. The x-coordinate of every point on the y-axis is 0. The y-coordinate of every point on the x-axis is 0. The origin is the point (0, 0). ‘The origin divides the x-axis into the positive x-axis to the right and the negative x-axis to the left. It divides the y-axis into the positive and negative y- axis above and below. The axes divide the plane into four regions called quadrants, numbered counterclockwise as in Fig. 6. Positive yanis 5 Cartesian coordinates. A Word About Scales When we plot data in the coordinate plane or graph formulas whose variables have different units of measure, we do not need (o use the same scale on the two axes. If we plot time vs. thrust for a rocket motor, for example, there is no reason to place the mark that shows 1 sec on the time axis the same distance from the origin as the mark that shows 1 Ib on the thrust axis, When we graph functions whose variables do not represent physical measure: ‘ments and when we draw figures in the coordinate plane to study their geometry and trigonometry, we try to make the scales on the axes identical. A vertical unit 2 Coordinates, Lines, and Increments 9 Fist ‘quadrant 4) 5 The points on the axes all have coordinate pairs, bbut we usually label them with single numbers. Notice the coordinate sign patterns in the quadrants. of distance then looks the same as a horizontal unit. As on a surveyor’s map or a scale drawing, line segments that are supposed to have the same length will look as if they do and angles that are supposed to be congruent will look congruent. Computer displays and calculator displays are another matter. The vertical and horizontal scales on machine-generated graphs usually differ, and there are corresponding distortions in distances, slopes, and angles. Circles may look like ellipses, rectangles may look like squares, right angles may appear to be acute or obtuse, and so on, Circumstances like these require us to take extra care in interpreting what we see, High-quality computer software usually allows you t0 compensate for such scale problems by adjusting the aspect ratio (ratio of vertical {o horizontal scale). Some computer screens also allow adjustment within a narrow range, When you use a grapher, try to make the aspect ratio 1, or close to Increments and Distance When a particle moves from one point in the plane to another, the net changes in its coordinates are called increments. They are calculated by subtracting the coordinates of the starting point from the coordinates of the ending point EXAMPLE 1 In going from the point A(4, the increments in the x- and y-coordinates are 3) (0 the point B(2, 5) (Fig. 7), Ax -4 5-(-3) =8 a Ay Definition An inerement in a variable is a net change in that variable. If x changes from.) t0 x3, the increment in x is Ane exp, EXAMPLE 2 From C(5, 6) to D(S, 1) (Fig. 7) the coordinate increments are 0 Ay=1-6=-5. a 7 Coordinate increments may be Positive, negative, or zero, Ax 10 Preliminaries Viewal + ba? 5, Ve Oe » »,| The distance between points in the plane is calculated with a formula that ‘comes from the Pythagorean theorem (Fig. 8). Distance Formula for Points in the Plane ‘The distance between P(x, ¥:) and Q(x, y2) is d= V(Ax)? + (Ay? = V2 = 11)? + 02 =) 8 To calculate the distance between Px, ys) and Ole, yo), apply the Pythagorean theorem to triangle PCO. EXAMPLE 3 a) The distance between P(—1, 2) and QG, 4) is VG-=CDP + G—3F = VF + OP = V0 = VF-5 = 25. ) The distance from the origin to P(x, y) is VE- OFF — OF = VE FY. a Graphs ‘The graph of an equation or inequality involving the variables x and y is the set of all points P(x, 9) whose coordinates satisfy the equation or inequality. EXAMPLE 4 Circles centered at the origin a) Ifa >0, the equation 2? +»? resents all points P(x, y) whose dis- tance frm the engin is /st-F 92 = at =o. These points ison the ile of radius a centered at the origin. This circle is the graph of the equation x+y? =a? Fig. 90). 1b) Points (x, y) whose coordinates satisfy the inequality x?+ y* 2, is the set of all numbers obiained by squaring numbers greater than or equal (0 2. In symbols, the range is {x7[x > 2) or {yly > 4) of [4, 00). EXAMPLE 3 Fanetion Domain (x) Range (y) (1.1 fo.) (20,0) (0,00) (90,0) U(0, 60) {0,00) [0,c0) (0.41 (0, 00) The formula y = JT =a? gives a real y-value for every xin the closed interval from —1 to 1. Beyond this domain, 1 ~ x? is negative and its square root is not a real number. The values of 1 ~ x? vary from 0 to 1 on the given domain, and the square roots of these values do the same. The range of v1 — x7 is (0, 1] ‘The formula y = 1/x gives areal y-value for every x except x = 0. We cannot divide any number by zero, The range of y = 1/x, the set of reciprocals of all nonzero real numbers, is precisely the set ofall nonzero real numbers. The formula y = V¥ gives a real y-value only if x = 0, The range of y = VF is [0, 00) because every nonnegative number is some number's square root (namely, its the square root of its own square) 20 Preliminaries 25 This circle is not the graph of a function y = fx); it fils the vertical line test ‘Computers and graphing ealeulators graph functions in much this way—by stringing together plotted points—and the same question arises. In y= V4=x, the quantity 4 — x cannot be negative. That is, 4 ~ x > 0, or x <4, The formula gives real y-values forall x < 4, The range of V4 —x is [0, 00), the set of all square roots of nonnegative numbers. Graphs of Functions ‘The graph of a function fis the graph of the equation y = f(x). It consists of the points in the Cartesian plane whose coordinates (x, y) are input-output pairs for f Not every curve you draw is the graph of a function. A function f can have only one value f(x) for each x in its domain, so no vertical line can intersect the ‘graph of a function more than once. Thus, a circle cannot be the graph of a function since some vertical lines intersect the circle twice (Fig. 25). Ifa is in the domain of 1 function f then the vertical line x =a will intersect the graph of fin the single point (a, f(a). EXAMPLE 4 — Graph the function y = x? over the interval [-2,2 Solution ‘Step 1: Make a table of 2y-pairs that satisfy the function rule, in this case the equation y = x? ‘Step 2: Piot the points (x, y) whose ‘Step 3: Draw a smooth curve through coordinates appear in the table the plotted points. Label the curve with its equation. How do we know that the graph of y =x? doesn’t look like one of these curves? x x 26 Useful graphs. Domain: (=, 2) Range: (0,2) Domain: =, 0) 4 (0, Range: 3 Fun fons 21 To find out, we could plot more points. But how would we then connect them? The basic question still remains: How do we know for sure what the graph looks like between the points we plot? The answer lies in calculus, as we will see in Chapter 3. There we will use a marvelous mathematical tool called the derivative to find a ccurve's shape between plotted points. Meanwhile we will have to settle for plotting points and connecting them as best we can, Figure 26 shows the graphs of several functions frequently encountered in calculus. It is a good idea to leam the shapes of these graphs so that you can recognize them or sketch them when the need arises. Domain: (-=, =) Range: (0. aan Domain: 2,0) 0,2) pata « Range: 2.0)0 (0,2) Domain: (-», =) Range: C=.) x L - at Domain: (0, =) Range: 10.) y= mx for selected Yalu of Domain: (2, =) Range: ==) 2 Preliminaries Sums, Differences, Products, and Quotients Like numbers, functions can be added, subtracted, multiplied, and divided (except Where the denominator is zero) to produce new functions. If fand g are functions, then for every x that belongs to the domains of both f and g, we define functions J+ —g, and fg by the formulas Cf +80) = fe) + 8) (f= 9) = f@)- a) (fads) = Fest) At any point of D(f)M D(g) at which g(x) #0, we can also define the function fig by the formula LV = £2 (where gta) £0 (é)¢ I= ¢ (x) #0), Functions can also be multiplied by constants: If¢ is a real number, then the function of is defined for all x in the domain of f by (cf) = ef (0. EXAMPLE fe 10, 00) g(x) = VT=2 (-00, 1] g(x) = 3VT—x (-00, 1) Uf + ae) = vet VT-x 10,1] = D(A) D(a) (fg) = VE-VT=e 0.) pe axel (0.1) GOO feo wn Ley £0 0 J fen te Lona 12 [ (0.0 (r= exces) bya wh fen on Da Composite Functions Composition is another method for combining functions. Definition Iffand g are functions, the composite function f o g (*fcircle g") is defined by (fay) = F@@)). ‘The domain of f 0 g consists of the numbers x in the domain of g for which .9(2) lies in the domain of f fo Fae) a 27 The relation of fog to g and f 28 (a) Symmetry about the y-axis. If (x.y) is on the graph, so is (~x, y).(b) Sym metry about the origin. If (x,y) is on the ‘graph, so is (-x, ~y). 3 Functions 23 ‘The definition says that two functions can be composed when the range of the first lies in the domain of the second (Fig. 27). To find (f 0 g)(x), we first find g(x) and second find f (g(x). To evaluate the composite function g o f (When defined), we reverse the order, finding f(x) first and then gf (x)). The domain of g 0 f is the set of numbers x in the domain of f such that f(x) lies in the domain of g ‘The functions fg and g o f are usually quite different. EXAMPLE 6 If f(x) = J and g(x) =x +1, find a) (fosvx) — b) @OAKe) —-&) (Fo fi) — @) (go.g)Ka). Solution Composite Domain a) (fogs) = fie) = Ve) = VAT (1,00) b) eo A) = ef) = fd += E+ [0.00) 9 (fo PG) = FO) = VFO) = VR =x" (0,00) @) (oR) =e) =e) +1=G4+)+1=x4+2 — Ror (-c0,00) To see why the domain of fog is [—1, co), notice that g(x) =x +1 is defined for al wal x but belongs to the domain of fony ifx +12 0, tha ist say, 7 Even Functions and Odd Functions—Symmetry A function y = f(x) is even if f(—x) = f(x) for every number x in the domain of f Notice that this implies that both x and —x must be in the domain of f- The function f(x) = x? is even because f(—x) = (—x)* FO). ‘The graph of an even function y = f(x) is symmetric about the y-axis Since f(—x) = f(x), the point (x, y) lies on the graph if and only if the point (—x, y) lies on the graph (Fig. 28a). Once we know the graph on one side of the axis, wwe automatically know it on the other side ‘A funetion y= f(x) is odd if f(—x) = —f() for every number x in the domain of f- Again, both x and —x must lie in the domain of f. The function (2) = x) is odd because f (=x) = (=x)? = =: ~ f(s). ‘The graph of an odd function y = f(x) is symmetric about the origin. Since f(x) = ~ f(x) the point (x,y) Ties on the graph if and only ifthe point (—x, —y) lies on the graph (Fig. 28b). Here again, once we know the graph of fon one side of the y-ais, we know it on both sides. cu 24 Preliminaries Piecewise Defined Functions ‘Sometimes a function uses different formulas on different parts ofits domain. One example is the absolute value function maf FER ‘whose graph is given in Fig. 29, Here are some examples. 29 The absolute value function. EXAMPLE 7 — The function 30 To graph the function y = f(x) shown here, we apply different formulas to ox x<0 f=) *, OsxZl xe is defined on the entire real line but has values given by different formulas depending ‘on the position of x (Fig. 30). a EXAMPLE 8 The greatest integer function ‘The function whose value at any number x is the greatest integer less than or equal 1o x is called the greatest integer function or the integer floor function. It is denoted (x), of, in some books, [x] or [[x]]. Figure 31 shows the graph. Observe that U9=1 10) [+12] = 10.2) [-03)=-1 [-2J= different parts of its domain (Example 7). 31 The graph of the greatest integer EXAMPLE 9 The least integer function ‘The function whose value at any number x is the smallest integer greater than or ‘equal to x is called the least integer function or the integer ceiling function. It is denoted [x]. Figure 32 shows the graph. For positive values of x, this function might represent, for example, the cost of parking x hours in a parking lot which charges $1 for each hour or part of an hour, i o* 2] function y = [x] lies on or below the line yaxs ‘provides an integer floor for x 32 The graph ofthe least integer function y ~ x) lies on or above the line y= x, 50 It provides an integer ceiling for x a Exercises 3 25 Exercises 3 Functions In Beeeises 1-6, find the domain and range of each function. 1 say 2 fw =t-Vi L 1 — 4 — a FIN= JZ, FO- In Brereises 7 and 8, which ofthe graphs are graphs of functions of ¥, and which are not? Give reasons for your answers. 1a) » t a * 7 * 8a) » x y a 7 a u Finding Formulas for Functions 9, Express the aca and petimeter of an equilateral angle as a function ofthe tangles side length x 10, Express the side length ofa square as a function ofthe length d ofthe square’s diagonal. Then express the area as a funtion of the diagonal length 11, Express the edge length of a cube as a function of the cube's diagonal length d. Then express the surface area and volume of the cube asa function of the diagonal length 12, A point P in the first quadrant lies on the graph of the function Fx) = Vi. Express the coordinates of P as functions of the Slope ofthe line joining P to the origin. Functions and Graphs Graph the functions in Exercises 13-24. What symmetis, if any, do the graphs have? Use the graphs in Fig. 26 for guidance, as needed. 13, y= x 1, y=-2 17. y= Visi 9. y=23/8 2 y=? Be y=C0 25, Graph the following equations and explain why they are not graphs of functions of x ®) bles » 26. Graph the following equations and explain why they are not _raphs of functions of x a) il +b1 by irtyi=l Even and Odd Functions In Exercises 27-38, say whether the function is even, odd, or neither. 2. fe) = 28. fon) = 2. f(x) 30. f(a tx 3h. as) 38. a6) 1 38. Ai) = = 31. A) = +1 38. ht) = 211+ Sums, Differences, Products, and Quotients In Bxercises 39 and 40, find the domains and ranges of fg, f +8 and f +g B f=H ge) = VERT 40, fe) = VEFT, gly = Vest In Bxercises 41 and 42, find the domains and ranges of J ¢.f/2. and g/f, 4 fe 2. fa) =, 26 Preliminaries Composites of Functions 43. If f(0) =x 45 and g(x) 3, find the following, a) f(s) ») sO) 9 fe) @) sf) 8 K-35) 0 ee) 8) se) bh) gle) 44 IF fe and g(x) = 1/(x + 1), find the following. a) f(gt1/2) by) (fay 2 fe) @) ef) © FF) 0 ee) 8) 0) h) gle) 4S. If u(x) = 4x 5, v(x) = 27, and f(x) = I find formulas for the following a) Mecsen)) by a(forry) 8 vucfery) @ (fue) 9 fluiotx)) D fou 46, IE f(x) = J%, ge) = 2/4, and A(x) = 4x — 8, find formulas: forthe following, a) (ets) by) L@eN”) 8 hse @) gfe) 9 flghey 1) Five) Let fx) =4—3, (2) = VE, h(x) =H, and f(x) = 20. Exe press each of the functions in Exercises 47 and 48 as a composite involving one or more of fg, /, and j Ma) y= VE-3 » Oo ya @ ° 0 48. a) » 3 @ ° 0 49, Copy and complete the following table. we fo (Fog) a) x-7 ve by x42 3x ° ves a ° x 1 ot x 50. A magic trick. You may have heard of a magic trick that goes like this: Take any number. Add S. Double the result. Suburat 6, Divide by 2. Subtract 2. Now tell me your answer, and I'l tell you what you started with Pick a number and try it ‘You can see what is going on if you let x be your original number and follow the steps to make a formula f(x) for the amber you end up with, Piecewise Defined Functions GGraph the functions in Exercises 51-4. Uses! syo=[f,, OSS 2 ew={) s Osx! siro=[}o" 25] _flm x<0 seom={ 550 5, Finda fom foreach function graphed. » » y , gH Tih 6, Find formula foreach function groped a) ») , : oe tree Lit 2 op T ‘The Greatest and Least Integer Functions 37. 58, 59. 60. For what values of x is (a) Lx] = 0? (b) [x] = 02 ‘What real numbers x satisfy the equation Lx) = [x1? Does [x] = Le forall eal «2 Give reasons for your answer. Graph the function afl #20 Dalle ese) Why is f(x) called the integer part of x? Even and Odd Functions a. ‘Assume that fis an even function, ¢ is an odd function, and both f and g are defined on the entire real line R. Which of the following (where defined) are even? odd? 4 Shifting Graphs 27 » © s/f Wi Grapher e 0 fee 63. (Continuation of Example 5) Graph the functions f(s) = JF ) a and g(x) = vI= together with their (a) sum, (b) product, 2 Cana fanetion be both even and odd? Give reasons for your _(@) wo ferences, (2) tvo quotient ae 64, Let f(x) =x ~7 and g(x) = 3°, Graph fand g togesher with fog and eof Shifting Graphs This section shows how to change an equation to shift its graph up or down or to the right or left. Knowing about this can help us spot familiar graphs in new locations. It can also help us graph unfamiliar equations more quickly. We practice ‘mostly with circles and parabolas (because they make useful examples in calculus), but the methods apply to other curves as well. We will revisit parabolas and circles in Chapter 9 » How to Shift a Graph To shift the graph of a function y = f(x) straight up, we add a positive constant to the right-hand side of the formula y = f(x). EXAMPLE 1 — Adding 1 to the right-hand side of the formula y = x? to get y =27 +1 shifts the graph up 1 unit (Fig. 33). a ‘To shift the graph of a function y = f(x) straight down, we add a negative constant to the right-hand side of the formula y= f(x). EXAMPLE 2 Adding —2 to the right-hand side of the formula y = x? to get \2 units y =x? 2 shifts the graph down 2 units (Fig. 33). a ‘To shift the graph of y = f(x) to the left, we add a positive constant to x 33 To shift the graph of flx) = x2 up (or down}, we add postive (or negative) EXAMPLE 3 Adding 3 to x in y = x2 to get y= (x +3) shifts the graph 3 constants to the formula for f units to the left (Fig. 34), a 34 To shift the graph of y = x to the left, we add a positive constant to x. To shift the graph to the right, we add a negative constant to x 28 Preliminaries preter rss 35 The graph of y =? shifted to three new positions in the xy-plane. P09) 36. A circle of radius a in the xy-plane, With center at (hk) To shift the graph of y = f(x) to the right, we add a negative constant to x. EXAMPLE 4 Adding -2 0.x in y = x? to get y = (x — 2)? shifts the graph 2 units to the right (Fig, 34), a po Shift Formulas \VeRricaL, SHIFTS Yyok= fo) or — Shifts the graph up k units if k > 0 y= f(x) +k Shifts it down [k| units if k <0 Hosizovrat. SHirTs =fa-h) Shifts the graph right h units fh > 0 Shifts it left [i] units if h < 0 EXAMPLE 5 The graph of y = (x ~ 2) ~2 is the graph of y =x? shifted 2 units to the right and 2 units down. The graph of y = (x +1)? +3 is the graph of y =. shifted 1 unit to the left and 3 units up (Fig. 35). a Equations for Circles A circle is the set of points in a plane whose distance from a given fixed point in the plane is constant (Fig. 36). The fixed point is the center ofthe circle: the constant distance isthe radius. We saw in Section 2, Example 4, that the circle of| radius a centered at the origin has equation x? + y* = a*. If we shift the circle to place its center at the point (h, &), its equation becomes (x — h)? + (y — k)? = a’. ‘The Standard Equation for the Circle of Radius a Centered at the Point 8) (WP + 0-8) EXAMPLE 6 If the circle x? + y? = 25 is shifted 2 units to the left and 3 units up, its new equation is (x + 2)? + (y — 3)? = 25. As Bq. (1) says it should be, this. isthe equation ofthe cirle of radius 5 centered at (hk) = (~2 3). a EXAMPLE 7 The standard equation for the circle of radius 2 centered at @G, 4)is (-3F +047 = OF (3 + -49 4 Shifting Graphs 29 ‘There is no need to square out the x- and y-terms in this equation. In fact, it is better not to do so. The present form reveals the circle’s center and radius. EXAMPLE 8 Find the center and radius of the circle (+0 +5) =3. Solution Comparing m+ -—w? with @ =P + OFF =3 shows that and a = V3. The center is the point (i, k) = (1. —5); the radius is a = V3. a Technology Square Windows We use the term “square window” when the units or scalings on both axes are the same. In a square window graphs are true in shape. They are distorted in a nonsquare window. ‘The term square window does not refer to the shape of the graphic dis- play. Graphing calculators usually have rectangular displays. The displays of Computer Algebra Systems are usually square. When a graph is displayed, the x-unit may differ from the y-unit in order to fit the graph in the display, resulting in @ distorted picture. The graphing window can be made square by shrinking or stretching the units on one axis to match the scale on the other, siving the true graph. Many systems have built-in functions to make the win- dow “square.” If yours does not, you will have to do some calculations and set the window size manually to get a square window, or bring to your viewing some foreknowledge of the true picture, ‘On your graphing utility, compare the perpendicular lines yy = x and ya=—x-+4 in a square window and a nonsquare one such a5 (—10, 10} by [10, 10]. Graph the semicircle y= V8— x? in the same windows. Two perpendicular lines and a semicircle graphed distorted by a rectangular window, If an equation for a circle is not in standard form, we can find the circle’s center and radius by first converting the equation to standard form, The algebraic technique for doing so is completing the square (see inside front cover). EXAMPLE 9 Find the center and radius of the circle vty +4r—6y-3=0. 30 Preliminaries Buerior (ci)? + =D? > a? wo. Imerior (x — )? + (y BP < at a 0 a 37 The interior and exterior of the circle eA Yk =a Venex at ongin 38 Besides determining the direction in ‘which the parabola y = ax? opens, the number a isa scaling factor. The parabola widens as a approaches zero and narrows as [a becomes large. Solution We convert the equation to standard form by completing the squares in wand y: yi tde—6y 3 =0 Gather terms. Monet constant tthe righthand (44x ) +07 -6y (°++())+(-+)) QE y —6y +9 =3-+4449 (424+ 9-3 = 16 Write each quadratic as 3 squared linear expression With the equation now in standard form, we read off the center's coordinates and. the radius: (h, &) = (=2,3) and a = 4, a Add the square of hal the ‘oefclent of to cach ‘de of the equation, Do the same for The parenthetical expressions fn the lefthand side are tox perfect squares (hare) + Interior and Exterior The points that lie inside the circle (x — h)? + (y — k= than a units from (h, &). They satisfy the inequality are the points less (WP $b? a EXAMPLE 10 Inequality Region x+y? <1 Interior of the unit circle YP <1 Unit cele plus its interior 24y?=1 Exterior of the unit circle x24)? 1 Unit circle plus its exterior a Parabolic Graphs The graph of an equation like y = 3x? or y = ~5x? that has the form year is a parabola whose axis (axis of symmetry) is the y-axis. The parabola’s vertex (point where the parabola and axis cross) Ties at the origin. The parabola opens upward if a > 0 and downward if a < 0. The larger the value of la], the narrower the parabola (Fig. 38). If we interchange x and y in the formula y = x say? . we obtain the equation symmetry pee oaenee Sr 2 ere at ovigin 39 The parabola x = ay? is symmetric about the x-axis. It opens to the right if >and to the left a <0 40 ‘The graphs of the functions y = VX and y = —¥x join at the origin to make the graph of the equation x= y! (Example 11) 41. The parabola y = ax’, a >0, shifted h Units to the right and k units up. 4 shifting Graphs 31 With x and y now reversed, the graph is a parabola whose axis is the x-axis and whose vertex lies at the origin (Fig. 39). EXAMPLE 11° ‘The formula x = y* gives x as a function of y but does nor give as a function of x. If we solve for y, we find that y = :k/z. For each positive value of x we get vo values of y instead of the required single value ‘When taken separately, the formulas y = /* and y = ~/% do define functions of x. Each formula gives exactly one value of y for each possible value of x. The raph of y = J is the upper half of the parabola x = y*. The graph of y = —/x is the lower half (Fig. 40). a The Quadratic Equation y = ax?+bx+c, a0 “To shift the parabola y ax? horizontally, we rewrite the equation as yar hy To shift it vertically as well, we change the equation to yok sale hy. @ ‘The combined shifts place the vertex at the point (/, &) and the axis along the Tine x=h (Fig. 4), Normally there would be no point in multiplying out the right-hand side of Eq, (2). In this case, however, we can learn something from doing so because the resulting equation, when rearranged, takes the form a? + be te. @ This tells us that the graph of every equation of the form y bxtc,a40, is the graph of y = ax’ shifted somewhere else. Why” Because the steps that take us from Eq. (2) to Bq, (3) can be reversed to take us from (3) back to (2). The curve y = ax? + bx +c has the same shape and orientation as the curve y = ax? ‘The axis of the parabola y = ax? + bx +c turns out tobe the line x = ~b/(2a), ‘The y-intercept, y = c, is obtained by setting x = 0. y ‘The Graph of y=ax?+br +e, a#0 ‘The graph of the equation y= ax?+bx+c,a #0, is 2 parabola. The parabola opens upward if a > 0 and downward if a <0. The axis is the line b Sl @ ‘The vertex of the parabola is the point where the axis and parabola intersect. Its x-coordinate is x= —b/2a; its y-coordinate is found by substituting x = =b/2a in the parabola’s equation. EXAMPLE 12 Graphing a parabola Graph the equation 32 Preliminaries jenexis(-1,2) > ven. 9) ranmmes\ | nee \! ena) Imerceptaty= 4 V (0,4 Timercepts at se dandx 42 The parabola in Example 12. Exercises 4 Shifting Graphs 1, Figure 43 shows the graph of y = —x? shifted to two new posi- tions, Write equations for the new graphs. 43 The parabolas in Exercise 1 Solution We take the following steps. Step 1: Compare the equation with y = ax? + bx + to identify a, b, and e. Step 2: Find the direction of opening. Down, because a <0. Step 3: Find the axis and vertex. The axis is the line b <) = =- ea 2a 3-1) so the x-coordinate of the vertex is —1. The y-coondinate is 9 n+4=3 The vertex is (~1,9/2). Step 4: Find the xintercepts (if any). 1 Set y = nthe ge ~etano oral ction 42n-8=0 Sve a6 ws (2044) 0 ra cand Step 5: Sketch the graph. We plot points, sketch the axis (lightly), and use what ‘we know about symmetry and the direction of opening to complete the graph ig. 42) Write equations for the new graphs. Position) \ froin) 44 The parabolas in Exercise 2. 2. Figure 44 shows the graph of y =x” shifted to two new positions. ‘3, Match the equations listed in (@)-(@) to the graphs in Fig. 45. a) y=Q-pi-4 by y=-2842 © yao 4242 a) y= (r+3"-2 Poston 2 Postion 1 45. The parabolas in Exercise 3 4. Figure 46 shows the graph of y = —° shifted to four new posi tions. Write an equation for each new graph, 23) ey 46 The parabolas in Exercise 4 [Exercises 5-16 tell how many units and in what directions the graphs of the given equations are (0 be shifted. Give an equation for the shifted graph. Then sketch the original and shifted graphs together, Exercises 4 33 labeling each graph with its equation. Use the graphs in Fig. 26 for reference as needed. 8. st4yt=49 Down 3, let 2 6 PH y?=25 Up 3 tees Tyas? Left 1, dowa 1 & y=? 9 y= VF Left ost 10, y=—VE Right 3 AL y=2x—7 Up? Right 1, down 1 saya Heb bs Downs it Bx 14, x =—3)* Up? tight 3 1B y 16. y= 1/x? Left 2, down 1 Graph the functions in Exercises 17-36. Use the graphs in Fig. 26 {or reference as needed. yP Let 1 Ix Up Ie right 1 11. y= vera 18, y= JOE 1. y= [x -21 20, y= Nx 2 y= JE=T yal-vE By=e4y Ly=-9"" yal yee HT 2 24. 26. yt 4x” 28 30, 38 y= 37. The accompanying figure shows the graph of a function f(x) with domain (0,2) and range (0, 1]. Find the domains and ranges ofthe following functions, and sketch theit graphs, a) fy +2 b) fo)-1 ©) 2f@) a =F) 2 fa+2 ) so=) 8) fo by -fle+D 41 34 Preliminaries ‘38, The accompanying figure shows the graph ofa function g(t) with domain [4,0] and range [-3, 0]. Find the domains and ranges Of the following functions, and sketch their graphs. a a) b) -2(9) 2 a +3 @ 1-80) e142) e-2 8) s-9 hy -@—4) Circles In Exercises 39-44, find an equation for the circle with the given center C(hk) and radius a, Then sketch the circle inthe sp-plane, Include the cicle’s center in your sketch, Also, label the eiele’s 2 and y-interceps, if any, with their coordinate pairs 3.C0,2), a=? 40. 6(-3.0), a 4 CO.S), 42 CLD), 0 43. c-V3, -2), 44.0,1/2, Graph the circles whose equations are given in Exercises 45-50. Label each circle’s center and intercepts (if any) with their coordinate pairs. 45, 8 ty? 4dr —dy 40 Sr t4y el 3y-4=0 4r— 9/4) ae t4y= 2 =3 Parabolas Graph the parabolas in Exercises 51-58. Label the vertex, axis, and Intercepts in each case woe 3 52 yar tay $4, ys? tar —5 $6. y=2xt x 43 3 ae bdr 85. yuna? -6r-5 ie ie Shysgvtatd SB y=— ph 4 2e+4 ‘59. Graph the parabola y of fx) = vee. (60. Graph the parabola y range of g(x) = v3= eax x2, Then find the domain and range ‘Then find the domain and Inequalities DDeseribe the regions defined by the inequalities and pairs of inequal- ities in Exerises 61-68, OL xe tyt>7 @vryes 6. w-pisyte4 6 EOD Ed Gxrtyel ety <4 O64 4 (EDP +Y 61 4 y46y <0, yo 3 OEY dr edy eh xd 69. Write an inequality that describes the points that lie inside the circle with center (~2, 1) and radius 6. 70, Write an inequality that describes the points that lie outside the circle with center (—4,2) and radius 4 ‘TA. Weite a pair of inequalities that deseribe the points that Ie inside fr on the circle with center (0, 0) and radius Y/2, and on or to the right ofthe vertical line through (1,0). ‘72. Weite a pair of inequalities that describe the points that lie outside the circle with center (0, 0) and radius 2, and inside the circle that has center (1, 3) and passes through the origin. Shifting Lines 73, The line y=mx, which passes through the origin, is shifted setcally and horizontally to pass through the pont (x, 9) Find an equation for the new line. (This equation is called the line's point-slope equation) 1A, The line y = max is shifted vertically to pass through the point (0,6). What is the new line's equation? Intersecting Lines, Circles, and Parabolas In Exercises 75-82, graph the two equations and find the points in Which the graphs intersect. 7. y=2x, Payad TB xty=l @-Ditytet Th y-x=1 y Bxey=0 y=-G-P yoo, y=dto1 y=}, yao? a etysh 84) © CAS Explorations and Projects In Exercises 83-86, you will explore graphically what happens to the raph of y = fax) as you change the value of the constant a. Use CAS or computer grapher to perform the following steps. a) Plotthe function y = f(x) together withthe function y for a= 3, and 10 over the specified interval. Deseribe what happens to the graph as a increases through positive values by Plotzhe function y = f(c)and y = f(ax) forthe negative values ‘a = ~2, 3. What happens to the graph in this situation? ©) Plot the function y= f(x) and y= fax) for the fractional values @ = 1/2, 1/3, 1/4. Describe what happens to the graph when ja} < 1 5 Trigonometric Functions 35 [=10, 10) (ax) BOD Radia | B 48 The angles of two common triangles, in degrees and radians, Trigonometric Functions This section reviews radian measure, trigonometric functions, periodicity, and basic trigonometric identities. Radian Measure In navigation and astronomy, angles are measured in degrees, but in calculus it is best to use units called radians because of the way they simplify later calculations (Section 2.4) Let ACB be a central angle in a unit eirele (circle of radius 1), as in Fig. 47 — 47 The radian measure of angle ACB is Tinea ‘the length of the arc AB, The radian measure @ of angle ACB is defined to be the length of the circular are AB, Since the circumference of the circle is 2x and one complete revolution of a circle is 360”, the relation between radians and degrees is given by the following equation, x radians 80° EXAMPLE 1 Conversions (Fig. 48) Convert 45° to radians: 45.2 F rad Convert © rad to degrees: 30° 36 Preliminaries Positive Negative ‘Conversion formulas ‘An angle in the xy-plane is said to be in standard position if its vertex lies at the origin and its initial ray lies along the positive x-axis (Fig. 49). Angles measured 1 degree = <5 (60.02) radians counterclockwise from the positive x-axis are assigned positive measures; angles measured clockwise are assigned negative measures, ‘When angles are used to describe counterclockwise rotations, our measurements can go arbitrarily far beyond 2x radians or 360°. Similarly, angles describing clockwise rotations can have negative measures of all sizes (Fig. 50). Degrees to radians: mult 7 res t0 ipl by 1 radian = 18° (ae 57) degrees Radians to degrees: multiply by = y y [50 Nonzero radian measures can be positive or negative. Gi ist i ice of al There is a useful relationship between the length s of an arc AB on a circle of radius rand the radian measure @ ofthe angle the arc subtends atthe etc’ 31, Teradan measure of angle ACB is enter (Fg. SI) If we draw a uit circle with the same center C, the are A’B" eee ae ene Cot by the angle will have length 8, by the definition of radian measure. From the found from any other circle as s/r. ilarity of the circular sectors ACB and A'CB', we then have s/r = 6/1 opposite aacent sings PR gac gz MP hyp co _ cost soo BP a no 22 epg aH ad opp 52 Trigonometric ratios of an acute angle. '53 The trigonometric functions of a {general angle # are defined in terms of x, yand © 54 The new and old definitions agree for ‘acute angles. 5 Trigonometric Functions 37 Radian Measure and Are Length or s=r8 Notice that these equalities hold precisely because we are measuring the angle in radians, Angle Convention: Use Radians From now on in this book it is assumed that all angles are measured in radians unless degrees or some other unit is stated explicitly. When we talk ‘about the angle 2 /3, we mean 2/3 radians (which is 60°), not x /3 degrees. When you do calculus, keep your calculator in radian mode. EXAMPLE 2 Consider a circle of radius 8. (a) Find the central angle subtended by an are of length 27 on the circle. (b) Find the length of an are subtending a central angle of 3/4, Solution a) The Six Basic Trigonometric Functions You are probably familiar with defining the trigonometric functions of an acute angle in terms of the sides of a right triangle (Fig. 52). We extend this definition to ‘obtuse and negative angles by first placing the angle in standard position in a circle of radius r We then define the trigonometric functions in terms of the coordinates of the point P(x, y) where the angle’s terminal ray intersects the circle (Fig. 53). Sine: sing = Cosecant: ese = 5 y Cosine: cos = * Secant seco = Tangent: tang => Cotangent: cot @ = * ‘These extended definitions agree with the right-triangle definitions when the angle is acute (Fig. 54) ‘As you can see, tan@ and sec6 are not defined if x = 0. This means they are 38 Preliminaries , not defined if @ is ++ /2, 37/2, .... Similarly, cot@ and esc are not defined for values of @ for which y =0, namely @=0, 7, +2 Notice also the following definitions, whenever the quotients are defined. Pla, 9) = (72088, rsin sind tand = no = a0 cos one = and 1 1 ecO=—= xe = = sO sb se Sind 55 The Cartesian coordinates of a point inthe plane expressed in terms of rand @.. The coordinates of any point P(x, y) in the plane can now be expressed in terms of the point’s distance from the origin and the angle that ray OP makes with the positive x-axis (Fig. 55). Since x/r = cos@ and y/r = sind, we have x= rcos8, yy =rsind, o Values of Trigonometric Functions If the circle in Fig. 53 has radius r = 1, the equations defining sind and cos » become Unitcirle cos? =x, siné=y, ‘We can then calculate the values of the cosine and sine directly from the coordinates of P if we happen to know them, or indirectly from the acute reference triangle made by dropping a perpendicular from P to the x-axis (Fig. 56). We read the magnitudes s of x and y from the triangle’s sides. The signs of x and y are determined by the ‘quadrant in which the triangle lies. EXAMPLE 3 Find the sine and cosine of 2/3 radia . Solution 56 The acute reference triangle for an Step 1: Draw the angle in standard position in the unit circle and write in the angle 8. lengths of the sides of the reference triangle (Fig. 57) 57 The triangle for calculating the sine and cosine of 2313 radians (Example 3). 5 Trigonometric Functions 39 Step 2: Find the coordinates of the point P where the angle’s terminal ray cuts the circle: | vos 2 1 — ates cos = s-coordinate of P = —5 Qn 3 sin = y-coordinate of P = “>. a tT 58 The CAST rule 59 The triangle for calculating the sine and cosine of =i radians (Example 4) | eye | ‘A useful rule for remembering when the basic trigonometric functions are positive and negative is the CAST rule (Fig. 58) EXAMPLE 4 Find the sine and cosine of —x /4 radians Solution Step 1: Draw the angle in standard position in the unit circle and write in the lengths of the sides of the reference triangle (Fig. 59) Step 2: Find the coordinates of the point P where the angle’s terminal ray cuts the circle: cos(~2) = comin of sin(—2) = yconinae of? = 22 Q Calculations similar to those in Examples 3 and 4 allow us to fill in Table 2. Table 2 Values of sin 6, cos 6, and tan for selected values of 6 Degrees a et ee) 45 oo 9 S80 O (radios) -x 3nd nf —nls 0k sin @ 2/0 ec eee 200) Oe 172 eee er aaa vin 0 cos 0 1 -V 0 Yn 1 Jn VR 1 0 -vIRR AL tan 8 o 1 “1 0 vB 1 3 -1 o 40 Preliminaries (60 The graphs of the six basic ‘trigonometric functions as functions of radian measure. Each function's periodicity shows clearly in its graph. Periods of trigonometric functions Period =: tan (x +) = tan Period 2: Domain: All real numbers except ol Jnteger multiples of 22 Range: (=, =) Domain: x # 0.2m. 22, Domain: x # 0, Range: =, “Hf, ) Range: =) Graphs ‘When we graph trigonometric functions in the coordinate plane, we usually denote the independent variable by x instead of 8. Sce Fig. 60. Periodicity When an angle of measure x and an angle of measure x-+ 2 are in standard posi- tion, their terminal rays coincide. The two anges therefore have the same trigono- metic values. For example, cos (r+ 27) = cos. Functions like the tigonometic functions whose values repeat at regular intervals are ealed perio Definition A function f(x) is periodic if there is a positive number p such that F(x+ p) = F(x) forall x. The smallest such value of p is the period of f AS we can sce in Fig. 60, the tangent and cotangent functions have period p = 7. ‘The other four functions have period 27. Figure 61 shows graphs of y = cos2x and y = cos(x/2) plotted against the graph of y = cos.x. Multiplying x by a number greater than | speeds up a trigono- metric function (increases the frequency) and shortens its period. Multiplying x by 4 positive number less than I slows a trigonometric function down and lengthens its period. 5 Trigonometric Functions 41 61. (@) Shorter period: cos2x. (b) Longer period: cos (2) 62. The reference triangle for a general angle @, The importance of periodic functions stems from the fact that much of the behavior we study in science is periodic. Brain waves and heartbeats are periodic, as are houschold voltage and electric current. The electromagnetic field that heats food in a microwave oven is periodic, as are cash flows in seasonal businesses and the behavior of rotational machinery. The seasons are periodic—s0 is the weather. ‘The phases of the moon are periodic, as are the motions of the planets. There is strong evidence thatthe ice ages are periodic, with a period of 90,000-100,000 years. Iso many things are periodic, why limit our discussion to trigonometric fune- tions? The answer lies ina surprising and beautiful theorem from advanced calculus that says tha every period function we want to use in mathematical modeling can bbe written as an algebraic combination of sines and cosines. Thus, once we learn the calculus of sines and cosines, we will know everything we need to know to ‘model the mathematical behavior of periodic phenomena. Even vs. Odd ‘The symmetries in the graphs in Fig. 60 reveal that the cosine and secant functions are even and the other four functions are odd: Even Oda Identities Applying the Pythagorean theorem to the reference right triangle we obtain by dropping a perpendicular from the point P(cosé, siné) on the unit cirele to the canis (Fig. 62) gives cos? 8 + sin? ‘This equation, true for all values of 8, is probably the most frequently used identity in trigonometry. a2 Dividing Eq. (2) in tum by cos?@ and sin? @ wives the ident ies ‘You may recall the following identities from an earlier cours. All the trigonometric identities you will need inthis book derive from Eqs. (2) and @) Angle Sum Formulas 0s (A+ B) = cos Acos B ~ sin Asin B B @) sin (A +.B) = sin Acos B + cos A ‘These formulas hold for all angles A and B. There are similar formulas for cos (4 — B) and sin (A — B) (Exercises 35 and 36). Substituting @ for both A and B in the angle sum formulas gives two more useful identities: Instead of memorizing Eqs. (3) you might find it helpful to remember Eqs. (4), and then recall where they came from. Double-angle Formulas cos 20 ® sin 26 =2 sin 6 cos Additional formulas come from combining the equations cos? 8 +-sin?® @ cos? 8 — sin? @ = cos 20. We add the two equations to get 2 cos? 0 = from the frst to get 2sin? @ = | — cos 28. + cos 20 and subtract the second nal Double-angle Formulas 1 +005 28 cos 28 2 cos? @ 6) sin? 6 © ‘When @ is replaced by 0/2 in Eqs, (5) and (6), the resulting formulas are called half-angle formulas. Some books refer to Eqs. (5) and (6) by this name as well. 63 ‘The square of the distance between A and B gives the law of cosines. Exercises S43 acos 0, in 8) The Law of Cosines If a, b, and c are sides of a triangle ABC and if @ is the angle opposite ¢, then +b? —2abcosd. a This equation is called the law of cosines. ‘We can see why the law holds if we introduce coordinate axes with the origin at C and the postive xaxis along one side of the triangle, as in Fig. 63. The coordinates of A are (b,0); the coordinates of B are (a cos 8, a sin 8). The square of the distance between A and B is therefore ¢ = (acs 6 ~ 6) + (asin 8)” = a°(cost @ + sin? @) +b — 2ab cos 8 i a+b? ~ 2ab cos 6. Combining these equalities gives the law of cosines, The law of cosines generalizes the Pythagorean theorem. If @ = 1/2, then cos = 0 and =a? +B Exercises 5 Radians, Degrees, and Circular Arcs 1. On a circle of radius 10 m, how long isan arc that subtends central angle of (a) 47/5 radians? (b) 110°? BA CALCULATOR 1 you oil & ameter whee frvard 30 im ove level ound, trough wht angel te wel ta? ‘ower in rans (othe nearest ety and eres (othe nearest degree), 2. A central angle ina circle of radius 8 is subtended by an are of Tength 10. Find the angle’s radian and degree measures. 3, CALCULATOR You want to make an 80° angle by marking an Evaluating Trigonometric Functions arc on the perimeter of # 12-in-diameter disk and drawing lines §, Copy and complete the table of function values shown on the from the ends ofthe are to the disk's center, To the nearest tenth following page. If the function is undefined at a given angle, of an inch, how long should the are be? center “UND.” Do not use a ealeulator or tbles. 44 Preliminaries tn} 0 wld sin 6 cos tan 8 cot 8 sec 8 ese 8 {6 Copy and complete the following table of function values. If the function is undefined at given angle, enter “UND.” Do not use ‘calculator o tables ° -39 nS —nl6 lk Sul sin cos 0 an @ cot @ sec 8 ese 0 In Exercises 7-12, one of sin x cos x, and tan x is given. Find the ther two if x lies in the specified interval 8 nx 9 Trigonometric Functions Graph the functions in Exercises 13-22, What is the period of each function? 13, sind 14, sin(x/2) 18. cos x cos 1S. 16. cos 11. ~sin™ 18, ~cos 2ax 19.0 («-3) 20, sin(s+ 2) an(eS)e1 mea(eeZ)- ia. Whar on of xk st Wi Sms et Beso ws eaue(%) B27. GRAPHER Sf Eaeana areca naa Slr oe ‘Comment on the behavior of ese. in relation to the sigas and values of sin. B28. GRAPHER Graph y= tanx and y= cotx together for —7 = x 7. Comment on the behavior of eotx in relation to the signs and values of tn x, 29, Graph y = sinx and y = [sin x} together. What are the domain and range of [sin.x|? 30, Graph y= sin and y = [sin together. What are the domain and range of [sin x]? Additional Trigonometric Identities ‘Use the angle sum formulas to derive the identities in Exercises 31-36 meo(-4) mon(er3) saaa(er$)aomr at aac 30, Wha happens ou te B= A ine ety eas(A— B) = you seat Laon? 38, Wa happens you ake B= in ean sum oma? Do the results agree with someth ig you already know? Using the Angle Sum Formulas In Exercises 39-42, express the given quantity in terms of sin x and 39, cos (+x) san (22-1) 43, Evaluate sin 7% 95 sin ( 2 40, sin x —x) ox (48) 313) a4 rants BE won (2-422) 46. Evaluate sin 2 Using the Double-angle Formulas Find the function values in Exercises 47-50. 47. cos S 48, cost 49. sia? 50, sin? = Theory and Examples S51. The tangentsum formula. The standard formula forthe tangent of the sum of two angles is ton A+ tan B tan (A +B) + Derive the formula, $2. (Continuation of Exercise 51.) Detive a formula for tan (A ~ B). 153, Apply the law of cosines to the triangle in the accompanying figure to derive the formula for cos(A ~ B). 54, When apie a iguresinilaro the nein Execs 3th aw 1 emins leads dit the fora fr co (A+B). Wha i that Heal ow dees he deraton Bo? B ss, cALcULATOR A wianle has sides a Cady Find te lenth of sie Bis, CALCULATOR A antes sides « =2 and b= 3 and angle Cay Find te lng of Side 6 51. The law of sins, ‘The law of snes say tht if Ban are the sides opposite the angles A.B and © in angle, then vind and b = 3 and angle sinB _sinC @ be Exercises 45 A “7 fy \h / fr Use the accompanying figures and the identy sin (x ~8 $in6, if required, to derive the law CALCULATOR A tingle has sides a = 2 and b = 3 and angle = 60" (as in Exercise 55). Find the sine of angle B using the law of sins. CALCULATOR A triangle has side ¢ = 2 and angles A= 1/4 and B= 7/3. Find the length @ ofthe side opposite A Biss, Eso, |. The approximation sin x~x. Its often useful to know that when x is measured in radians, sin * x for numerically small, values of x In Section 3.7, we will see why the approximation holds. The approximation errors less than 1 in $000 if | < 0.1 8) With your grapher in radian mode, graph y = sin.x and _y =x together in a viewing window about the origin. What 4o you see happening as x nears the origin? b) With your grapher in degree mode, graph y =sin.x and y= together about the origin again. How is the picture ‘different from the one obtained with radian mode? 9) A quick radian mode check. Is your calculator in radian mode? Evaluate sinx at a value of x near the origin, say x= 011. If sinx © x, the calculator is in radian mode; if not, it isn't. Try it General Sine Curves Pewee loving oe how ep or sie pu =Ase(%a—0) +0, where [Al is the amplitude, |B| is the period. C is the horizontal shift, and D is the vertical shift. entity A, B, C, and D for the sine Tunctions in Exercises 61-64 and sketch their graphs. oy 2sin(x +m) —L 64 The general sine curve Y= Asin [2r/BMe — O40, shown for A, B, C and D positive. The Trans-Alaska Pipeline ‘The builders ofthe Trans-Alaska Pipeline used insulated pad to keep the feat from the hot oil in the pipeline from melting the permanently frozen soil beneath. To design the pads, it was necessary to take into ‘account the variation in air temperature throughout the year, Figure 665 shows how we can use a general sine function, defined in the introduction to Exercises 61-64, to represent temperature data. The data points in the figure are plots of the mean air temperature for Fairbanks, Alaska, based on records of the National Weather Service from 1941 to 1970, The sine function used to fit the data is fe = 37sin( ~ 10») +25 where fis temperature in degrees Fahrenheit and x is the number of the day counting fromthe beginning ofthe year. The itis remarkably 0 65. Temperature in Fairbanks, Alaska. Find the (a) amplitude, (b) period, (c) horizontal shift, and (4) vertical shift of the general sine Function a J¢0) = ssn (Fets— 109) +25 shit(D) Amplitude (4) This ais isthe ine y= D. "This distance is the period. 66, Temperature in Fairbanks, Alaska. Use the equation in Exer cise 65 to approximate the answers to the following questions about the temperature in Fairbanks, Alaska, shown in Fig. 65, Assume that the year has 365 days a) What are the highest and lowest mean daily temperatures, shown? 1b) What is the average of the highest and lowest mean daily temperatures shown? Why is this average the vertical shift of the funtion? © CAS Explorations and Projects In Exercises 67-70, you will explore graphically the general sine function 2 faye Asin( ze 0) D 2 you change the valves ofthe constants A, B, C, and D. Use CAS or computer graphar to perform the steps in the exercises 67. The period 8. Set the constants A= 3,C = D =0. a) Pot f(x) forthe values B = 1, 3, 2, Sw over the interval « 65. Normal mean air temperature at e Fairbanks, Alaska, plotted as data points. E a ‘The approximating sine function s 5 2x & #09) = 375in ( 2 20 es) = 3739 (Fel 108)) +25 : (ource: "ls the Curve of Temperature Variation a Sine Curve?” by 8. M. Lando ‘and C. A, Lando, The Mathematics Teacher, 7:5, Fig. 2, p. 535 [September 1971) Mar Apr May Jon Tul Aug Sep Ost Nov Dec Jan Feb Mar 68. 1 10. nL 2 PRELIMINARIES 4x < x < dz, Describe what happens tothe graph of the general sine function as the period increases. b) What happens to the graph for negative values of B Tey it with B= —3 and B= ~2r The horizontal shift C. Set the constants A = 3, B = 6, D =0, 48) Plot (x) forthe values C= 0,1, and 2 over the interval yy 0), 80.0) Pp a Atay b) When is OP perpendicular to AB? Functions and Graphs 3. Ate there two functions f and g such that fo g reasons for your answer, o f? Give 4. Ate there two functions f and g withthe following property? The ‘graphs of fand g are not straight Tines but the graph of fo g is ‘a straight line. Give reasons for your answer, 5. IF f(a) is odd, can anything be ssid of g(x) = fe) ~ 22 What if Fis even instead? Give reasons for your answer, 6 If g(x) is an odd function defined for al values of x can anything be suid about ¢(0)? Give reasons for your answer, 1 8. Graph the equation y + |y| =x + [xh Ite. Graph the equation | + [y Trigonometry In Exercises 9-14, ABC is an arbitrary range with sides a, Band c opposite angles A,B, and C, respectively. 9. Find bia = V3, A= 2/3, B= 2/4. 10, Find sin B ifa=4,b=3,A=a/4, UL, Find cos A if a =2,b =2,¢ =3. 12, Find cifa=2,b 1A 13, Find sin B if a =2,b 14, Find sin€ if a =2,b= 4, =5 Derivations and Proofs 15, Prove the following identities. 1 wy Lesose sins Bins ~ Teoee L-cose Treosx "2 50 Preliminaries 16, Explain the following “proof without words” of the law of cosines, (Source: "Proof without Words: The Law of Cosines” Sidney H, Kung, Mathematics Magazine, Vol. 63, No.5, Dee. 1990, p. 342.) 17, Show thatthe area of triangle ABC is given by (/2)absinC = (1/2)besin A = (1/2)ca sin B. © a = B 18 Show that the area of triangle ABC is given by VS = ays = BS =O) where 5 = (a+ b-+0)/2 is the semi perimeter of the triangle.” 19, Properties of inequalities. Ifa and b are real numbers, we say that ais less than and write a = b if (and only if) b—a is 20, Properties of absolute values. Prove the following properties of absolute values of real aumbers. a) |-al/=la) 2) _ lal » Elsa 21, Prove thatthe following inequalities hold for any real numbers amdb 8) [al = bl if and only ita? < 9? ‘by lab] > Ilal ~ (I) 22, Generalizing the triangle inequality. Prove by mathematical induction thatthe following inequalities hold for any real mum bers ay, 3... +dy. (Mathematical induction is reviewed in Ap- pendix 1.) a) lay ag teal S la lag + lag) Dy fay ag Ho eal & lal = lal --+ = ll 23, Show that if fis both even and od then f(x) =: inthe domain of f 24, a) Even-odd decompositions. Let fbe a function whose do- main is symmetric about the origin, that is, ~x belongs 10 the domain whenever x does. Show that fis the sum of an ‘oven function and an odd function: FO) = EG) +00, ‘where Eis an oven function and O is an odd function. (Hint Let E(x) = (f(x) + f(=2))/2. Show that E(—2) = E(2), so that Eis even. Then show that O(x) = f(x) ~ E(x) is odd) 1b) Uniqueness. Show that there is only one way to write fas the sum of an even and an odd function. (Hint: One way is given in part (a). If also f(x) = is even and O} is odd, show that E ~ E) use Exercise 23 t0 show that E = &, and O for every positive. Use this definition to prove the following properties of 8% Grapher Explorations—Effects of Parameters inequalities. Ifa, , and ¢ are real numbers, then: ach = atccbte axb = a-c0 => ac be 2e) “Asterisk denotes more challenging problem, 25, What happens to the graph of y = ax? + be +e as 8) a changes while band c remain fixed? b) b changes (a and c fixed, a #0)? ©) ¢ changes (a and b fined, a #0)? 26, What happens to the graph of y = a(x +6)" be as 8) a changes while band c remain fixed? b) DB changes (a and c fixed, a #0)? ©) e changes (a and b fied, a #0)? 27. Find all values of the slope ofthe line y = mx +2 for which the -vintercept exceeds 1/2. Limits and Continuity OVERVIEW The concept of limit of a function is one of the fundamental ideas that distinguishes calculus from algebra and trigonometry. In this chapter we develop the limit, fist intuitively and then formally. We use limits to describe the way a function f varies. Some functions vary continuously; small changes in x produce only small changes in (x). Other functions ean have values that jump or vary erratically. We also use limits to define tangent lines to gtaphs of functions, This geometric application leads at once to the important concept of derivative ofa function. The derivative, which we investigate thoroughly in Chapter 2, quantities the way a function’s values change: Free fall Near the surface of the earth, all bodies fll withthe same constant acceleration. The distance a body falls after itis released from rest isa constant multiple ofthe square of the time elapsed, AC leas, that is what hhappens when the body falls in a vacuum, ‘where there is no air to slow it down, The square-of time rule also holds for dense, Ineavy objects like rocks, ball bearings, and steel tools during the first few seconds of ther fll ehrough air, before their velocities build up to where air resistance begins to matter. When air resistance is absent or insignificant and the only foree acting on @ falling body is the force of gravity, we call the way the body falls fre fal. Rates of Change and Limits In this section we introduce two rates of change, speed and population growth, This leads to the main idea of the section, the idea of limit. Speed ‘A moving body's average speed over any particular time interval isthe amount of distance covered during the interval divided by the length of the interval EXAMPLE 1A rock falls from the top of a 150-ft cliff. What is its average speed (a) during the first 2 see of fall? (b) during the 1-sec interval between second and second 2? Solution Physical experiments show that a solid object dropped from rest to fall freely near the surface of the earth will fall y= 16 fe during the first ¢ sec. The average speed of the rock during a given time interval is the change in distance, Ay, divided by the length of the time interval, Ar Ay _ 16(2)? — 16(0)? ft 2) Feri fs 2 Y= OPA IO? ge =e 52 Chapter 1: Limits and Continuity Table 1.1 Average speeds over short time intervals Ay _16(tp +h)? — L640? Average speed: 2” = A ‘Average speed over Average speed over Interval of length f Interval of length starting at f= 2 80 656 64.16 64.016 64.0016 EXAMPLE 2 Find the speed of the rock at ¢ = I and 1 = 2 sec. Solution We can calculate the average speed of the rock over a time interval Uo, fo +h}, having length Ar = h, as +h)? ~ 1652 rane h We cannot use this formula to calculate the “instantaneous” speed at fy by sub- stating h = 0, because we cannot divide by zero. But we can use it to calculate average speeds over increasingly short time intervals stating ati = 1 and fy = 2 When we do so, we see a pattem (Table 1.1) “The average speed on intervals starting at = 1 seems to approach a im: iting value of 32 a the length of the interval decreases, This suggests thatthe rock is falling at a speed of 32 fusec at tp = 1 sec. Similarly, the rock's speed at fy =2 see would appear to be 64 fuse. a Average Rates of Change and Secant Lines Given an arbitrary function y = f(x), we calculate the average rte of change of| ¥y with respect to x over the interval (x, x3] by dividing the change in value of y, Ay = f(x) — f(x), by the length of the interval Ax ‘hover which the change occurred, Definition ‘The average rate of change of y = f(x) with respect to x over the interval bax is Ay _ fla) = fe) _ Sort W = fe) Bx am h 7 Notice that the average rate of change of f over [x1,.x2] is the slope of the line through the points P(x, f(xi)) and QC, f(42)) (Fig. 1.1). In geometry, a line Joining two points of a curve is called a secant to the curve, Thus, the average rate ‘of change of f from x; to x2 is identical with the slope of secant PQ. Geometrically, an average rate of change is a secant slope. 1.1. Rates of Change and Limits 53 Pay foxy) 41.1. secant to the graph y = f(x). Its slope is ‘Ay/Ax, the average rate of change of f over the interval bx, #2. Experimental biologists often want to know the rates at which populations grow under controlled laboratory conditions. EXAMPLE 3 The average growth rate of a laboratory population Figure 1.2 shows how a population of fruit flies (Drosophila) grew in a SO-day experiment. The number of fies was counted at regular intervals, the counted values plotted with respect to time, and the points joined by a smooth curve. Find the average growth rate from day 23 to day 45. Solution There were 150 flies on day 23 and 340 flies on day 45, Thus the num- ber of flies increased by 340 — 150 = 190 in 45 —23 = 22 days. The average rate of change of the population from day 23 to day 45 was 4p _ 340-150 _ 190 ar 45-23 ~ 22 ‘Average rate of change: 86 flievday. ‘This average is the slope of the secant through the points P and Q on the graph in Fig. 1.2. The average rate of change from day 23 to day 45 calculated in Example 3 does not tell us how fast the population was changing on day 23 itself, For that we need to examine time intervals closer to the day in question. > 350] 00] 2s0| 200] 150 100 | 50 Number of es 1.2 Growth of a fruit fly population in a controlled experiment. (Source: Elements of Mathematical ° Biology by A.J. Lotka, 1956, Dover, New York, p. 69.) ‘Time (ys) 54 Chapter 1: Limits and Continuity 2 (45, 340) (40,330) 25,310) 0, 268) Slope of PQ = A pid ¢ (iesiday) 340 — 150 a3 320-150 1.3 The positions and siopes of four secants through the point P on the fruit fiy graph. 108,390) 7 7 | LAT "ote 340) Number of ies o eA SSCS C40) ‘Time (days) EXAMPLE 4 How fast was the number of flies in the population of Example 3 growing on day 23 itself? Solution ‘To answer this question, we examine the average rates of change over increasingly short time intervals stating at day 23. In geometric terms, we find these rates by calculating the slopes of secants from P to Q, for a sequence of points Q approaching P along the curve (Fig. 1.3) The values in the table show that the secant slopes rise from 8.6 to 16.4 as, the coordinate of Q decreases from 45 to 30, and we would expect the slopes to rise slightly higher as 1 continued on toward 23. Geometrically, the secants rotate about P and seem to approach the red line in the figure, a line that goes through P in the same direction thatthe curve goes through P. We will see that this line is called the tangent 0 the curve at P. Since the line appears to pass through the points (14, 0) and (35, 350), it has slope 350~0 Ss 16.7 fies/day approximately). 4 On day 23 the population was increasing at a rate of about 16.7 liesday. OI ‘The rates at which the rock in Example 2 was falling at the instants ¢ = 1 and f= 2 and the rate at which the population in Example 4 was changing on day 1 = 23 are called instantaneous rates of change. As the examples suggest, We find instantaneous rates as limiting values of average rates. In Example 4, we also pictured the tangent line to the population curve on day 23 as a limiting position ‘of secant lines. Instantaneous rates and tangent lines, intimately connected, appear in many other contexts. To talk about the two constructively, and to understand the connection further, we need to investigate the process by Which we determine limiting values, or limits, as we will soon call them. Limits of Function Values Before we give a definition of limit, let us look at another example. EXAMPLE 5 — How does the function f(x) 1 behave near x = 1? Tor ox 114. The graph of fis identical with the Tine y = x-+1 except at x = 1, where fis, not defined, 1.1. Rates of Change and Limits 55 Solution The given formula defines f for all real numbers x except x = 1 (we cannot divide by zero). For any x #1 we can simplify the formula by factoring the numerator and canceling common factors: (x= Dat) Lo) x1 for x41 x= ‘The graph of fis thus the line y =x +1 with one point removed, namely the point (I, 2). This removed point is shown as a “hole” in Fig. 1.4. Even though f(1) is not defined, it is clear that we can make the value of /(x) as close as we want to 2 by choosing x close enough to 1 (Table 1.2). We say that /'(x) approaches arbitrarily close to 2 as x approaches 1, or, more simply, f(x) approaches the limit 2 as x approaches 1. We write this as xo fin oT a lim fox) Table 1.2 The closer x gets to 1, the closer f(x) = (x? — 1)/tx ~ 1) seems to get to 2. Values of x Delow and above 1 09 La 099 Lol 0.599 1.001 0.999999 1.000001, Definition Informal Definition of Limit Let f(x) be defined on an open interval about xo, except possibly at xp itself. If f(x) gets arbitrarily close to L. for all x sufficiently close t0.x9, we say that f approaches the limit L as x approaches xy, and we write This definition is “informal” because phrases like arbitrarily close and sufficiently close are imprecise; their meaning depends on the context. To a machinist man. tufacturing a piston, close may mean within a few thousandths of an inch, To an astronomer studying distant galaxies, close may mean within a few thousand light ‘years. The definition is clear enough, however, to enable us to recognize and evaluate limits of specific functions. We will need the more precise definition of Section 1.3, however, when we set out to prove theorems about limits. 56 Chapter 1: Limits and Continuity 1.5 fim (4) = firm g6e) = fim AO) = 2. (2) Kent function (©) Constant funtion 1.6 The functions in Example 8, © he) e+ EXAMPLE 6 The existence of a limit as x —> Xo does not depend on how the function may be defined at x9, The function f in Fig 1.5 has limit 2 as x — 1 even though f is not defined at x = 1. The function g has limit 2 as x 1 even though 2 # (1). The function h is the only one whose limit as. x —> 1 equals its value at x = 1. For h we have lim.) h(x) = h(1). This kind of equality of limit and function value is special, and we will return to it in Section 1.5, a Sometimes lim,-.., f(x) can be evaluated by calculating (xo). This holds, for ‘example, whenever f(x) is an algebraic combination of polynomials and trigono- metric functions for which f (zo) is defined. (We will say more about this in Sections 12and 1.5.) EXAMPLE 7 a) lim @)=4 b) tim.) © lim x 4) lim (x—3)=10-3=7 _axt4 644 2 ©) lim, wenn g$3 245° 3 Q EXAMPLE 8 a) If fis the identity function f(x) = x, then for any value of xo (Fig. 1.6a), lim #0) = lim x =x. b) If fis the constant function f(x) = k (function with the constant value k), then for any value of xo (Fig. 1.6b), lim f(x) = lim k= a ‘Some ways that limits can fail to exist are illustrated in Fig. 1.7 and described in the next example, Exercises 1.1. 57 (o Unitstep funtion Uo 1.7. The functions in Example 9 wen (ofa) EXAMPLE9 A function may fail to havea limit ata point in its domain. Discuss the behavior of the following functions as x > 0. a) Ue) lie 5 b) wo= {i BY 0 x<0 a ron (ne oy Solution 8) Itjumps: The unit step funetion L(x) has no limit as x — O because its values, jump at x =O. For negative values of x arbitrarily close to zero, U(x) = 0. For positive values of x arbitrarily close to zero, Utx) = 1. There is no single value L approached by U(x) as x ~» 0 (Fig. 1.70. ) It grows too large: g(x) has no limits as x > 0 because the values of ¢ grow arbitrarily large in absolute value as x -> 0 and. do not stay close to any real number (Fig. 1.7b). ©) It oscillates too much: f(x) has no limit as x -> 0 because the function's values oscillate between +1 and —I in every open interval containing 0. The values do not stay close to any one number as x ~> 0 (Fig. 1.70). a Exercises 1.1 li its from Graphs 1. For the function g(x) graphed here, find the following limits or explain why they do not exis. a) lim gt) by tim gc) ©) im g(x) 58 Chapter 1: Limits and Continuity 2, For the function f(¢) graphed here, find the following limits or explain why they do not exist a) tim, fn) b) tim fo) © tim f( 3. Which of the following statements about the function y = f(x) sraphed here are wue, and which are false? 8) tim f(x) exists by lim fox) =0 © lim fay=1 @) tim foy=1 © lim fo) =0 lim f(x) exists at every pointy in (—1, 1) Which ofthe following statements about the function y = f(x) ‘graphed here are tue, and which are false? tyes 2 i $0) ders a is 0 lig 0) =2 iy fee oe a © at fed cine tcmry pete 3 Existence of Limits In Exercises $ and 6, explain why the limits do not exist, lim * 7. Suppose that a function f(x) is defined for all real values of x except x =o, Can anything be said about the existence of Tim,-+», £(0)? Give reasons for your answer 8. Suppose that a function f(x) is defined for all x in {1,1} Can anything be said about the existence of lim,g f(x)? Give reasons for your answer, vi F(8) =5, must f be defined at x= 17 1 it is, must 5 Can we conclude anything about the values of f at = 1? Explain 10. If f(1) = 5, must fim...) f() exist? IF it does, then must lim,..: f() = 57 Can we conclude anyrhing about lira, £8)? Explain. Calculator/Grapher Exercises—Estimating Limits 1, Let f(x) = (x2 — 9)/tr +3). Ha) CALCULATOR Make a table ofthe values of fat the points = 3.1, 301, 3.001, and so on as fr as your calcula tor ean go. Then estimate lim. (2). What estimate do you arrive at if you evaluate fat x = ~2.9, ~2.99, -2.999, instead? >) GRAPHER. Support your conclusions in (a) by graphing f near t= ~3 and using ZOOM and TRACE to estimate values on the graph as x > —3 ©) Find lim... f(2) algebraically 12, Let g(x) = (x? —2)/¢x - V2). a) CALCULATOR Make a table of the values of g atthe points = 14,14, 1.414, and so on though successive decimal approximations of V/2. Estimate lim,..y3_ (3) GRAPHER Support your conclusion in (a) by graphing ¢ rear y= v2 and using ZOOM and TRACE to estimate y-values on the graph as x —+ V2. © Find lim, g(2) algebraically. 13, Let G(x) = (+6)/(a? + 4x — 12) a) CALCULATOR Make 2 table of the values of G at x= =5.9, -$.99, -5.999.... Then estimate im... GC) What estimate do you ative at if you evaluate Gat x = 6.1, ~6001, ~6.001, ... instead? GRAPHER Support your conclusions in (a) by graphing G and using ZOOM and TRACE to estimate y-values on the sraph as x > -6. © Find lim,..¢ G(x) algebraically 14, Let h(x) = («2 ~ 28 ~ 3)/(82 = 4 43) a) CALCULATOR Make a table of the values of ft at x= 2.9,2.99, 2.999, and so on. Then estimate limy..s (3) ‘What estimate do you arrive at if you evaluate h at x= 3.1,3.01, 3001, . instead? Bn) Hp) Bb) GRAPHER Support your conclusions in (a) by graphing ‘a near x =3 and using ZOOM and TRACE to estimate y-values on the graph as x > 3 ©) Find tim, .5- (x) algebraically. 15, Let F(x) = ( — D/(leI =D. Ba) CALCULATOR Make tables of the values of f at values of that approach zy = —1 from above and Below. Then estimate lim f) Bb) GRAPHER Support your conclusion in (a) by graphing f near to = ~1 and using ZOOM and TRACE to estimate ‘yevalues on the graph as x —> —1 ©) Find lim...) f(s) algebraically 16, Let F(x) = (29 438 429/02 ~ |aD. Bay CALCULATOR Make tables of values of Fa vals of x that approach g = ~2 from above and below. Then estinaie lima FO) 8b) GRAPHER Support your conclusion in () by graphing F hd sing ZOOM and TRACE to estimate yrvalee on the gragh as x ~2. ©) Find im, > Fes algebra. 17, Let (6) = indy la) CALCULATOR Make ubles of values of ga ales of @ that approach =O em above an blow: Then extinate time 8) 1) GRAPHER. Suppo your conclusion in (a) by saphing ¢ neat 18 Le Gl) = (cos a) CALCULATOR Make bles of values of G a values of ¢ that approach fp = 0 from above and below. Then estimate Jim» G0), ) GRAPHER Support your conclusion in (a) by graphing G 19, Let f(x) = x40", a) CALCULATOR Make tables of values of fat values of x that approach x9 = 1 from above and below. Does f appear (0 hhave a limit as x —> 1? If 30, what is it? If not, why not? b) GRAPHER Support your conclusions in (a) by graphing f 20, Let f(x) = G' — 1/x. Ba) CALCULATOR Make tables of values of f at values of x that approach x = 0 from above and below. Does f appear to have a limit as x —> 07 If so, what is i? If not, why not? GRAPHER Support your conclusions in (a) by graphing f Hp) Limits by Substitution In Exercises 21-28, find the limits by substitution. Support your an swers with a grapher or calculator if available. 21 im 22 22, lim 2x Exercises 1.1 59 23. ig, xD) 3 25. tim 3x(2¢—1) 26, im, = je 2. im, xsine 28. ty SO Average Rates of Change In Exercises 29-34, find the average rate of change of the function over the given interval or intervals. 30. gx) (I-11, ()[-2.0) 31. h(t) = cotrs (@) (2/4, 32/4), () (9/6. /2] 32 g(t) =2 +080; @) 10,2), ) [2,71 33. RO) = VHF: (0,2) 34, PO) = 0 — 404.50: 11,2] 3S. Figure 1.8 shows the timeo-distance graph for a 1994 Ford ‘Mustang Cobra accelerating from a standstill a) Estimate the slopes of secants PQ), PQs, PQs, and PQs, arranging them in order in a table, What are the appropriate units for these slopes? b) Then estimate the Cobra’s speed at time ¢ 20 see. 650 oo 00 Distance (m) 200 100 epee gua gees ee 0 lapsed time (see) 1.8 The time-to-distance graph for Exercise 35. 60 chapter 1: Limits and Continuity 36. Figure 1.9 shows the plot of distance fallen (m) vs. time for a ©) Use your graph to estimate the rte at which the profits were ‘wrench that fll from the top platform of a communications mast changing in 1992, ee 38, CALCULATOR Make a table of values for the function F(x) = 48) Estimate the slopes of the secants PQs, PQs, PQs, and (&-+2)/(4 —2) at the points x =2,x = 11/10,x = 101/100, PQ4, arranging them in a table like the one in Fig, bb) About how fast was the wrench going when it hit the root? x = 1001/1000, x = 10001/10000, and x = 1 4) Find the average rate of change of F(x) over the imervals {yal foreach x #1 in your tbl by) Extending the table if necessary, ty to determine the rate of change of F(x) atx = 1 B39, CALCULATOR Let g(x) = vi for x > 0. 8) Find the average rte of change of g(x) with respect to x cover the intervals (1,2), [1, 1.5], and [1,1 +h]. 1) Make a table of vanes ofthe average rate of change of with respect tox over the iterval 1, 1+] for some vas of approaching 2eo, say h = 0.1, 0.01, 0.01, 0.001, 0.000 and 0.000001 ©) What does your tbl indicat isthe rate of change of g(8) with respect ot at r= 1? 4) Calculate the limit as approaches zero of the average rote of change of g(x) with respect to x over the interval 01+}. B40, carcutator Let fa) a) Find the average rate of change of f with respect t0 r over the interval () from ¢ = 2 to ¢ =3, and (i) from ¢ =2 10 b) Make a table of values of the average rate of change of f with respect to over the interval (2, 7), for some values of approaching 2, say T = 2.1, 2.01, 2.001, 2.0001, 2.00001, ‘and 2.000001, ©) What does your table indicate is the rate of change of f with, respect to fat ¢ = 2? 4) Calculate the limit as T approaches 2 of the average rate of change off with respect 0 £ over the interval from 210 T: Elapsed time (se) ‘You will have 10 do some algebra before you can substitute T=2. 1/t for #0. Distace fallen (im) 1.9 The time-to-distance graph for Exercise 36. © CAs Explorations and Projects 1 37. CALCULATOR The profits of a small company for each of a eemBany for cach oF Me In Exercises 41-46, use a CAS to perform the following steps: fist five yeas ofits operation are given in the following table: a) Plot the function near the point xy being approached, Year Profit in $1000, b) From your plot guess the value of the limi evr" ©) Evaluate the limit symbolically. How close was your guess? 1990 6 1991 2 a 1992 @ 1993 in 2, 1994 74 8) Plot points representing the profit as a function of year, and 43. Join them by as smooth a curve as you can 1b) What is the average rate of inerease of the profits between 1 cose 2x? 1992 and 1994? *) ysinx 3) 3 Scosx 1.2. Rules for Finding Limits 61 Rules for Finding Limits ‘This section presents theorems for calculating limits, The first three let us build ‘on the results of Example 8 in the preceding section to find limits of polynomials, rational functions, and powers, The fourth prepares for calculations later in the text Limits of Powers and Algebraic Combinations Theorem 1 Properties of Limits ‘The following rules hold if lim... f(x) = L and lim... g(x) = M (Land ‘M real numbers). 1. Sum Rule: Jim [f@) + 2@))=L+M 2. Difference Rule: tim [f(x) = g(x) = L-M 3. Product Rule: tim f(s) g6) =L-M 4. Constant Multiple Rule: Yim kf(x) = kL (any number &) x) & cui ste in LE argo 6 Power Rule: If m and mare integers, then lim [fey = L", provided L"/" is a real number. In words, the formulas in Theorem 1 say: 1. The li 2. The limit of the difference of two functions is the difference of their limits. 3. The limit of the product of two functions is the product of their limits, 4, The limit of a constant times a function is that constant times the limit of the function, 'S. The limit of the quotient of two functions is the quotient of their limits, provided the limit of the denominator is not zero. 6. The limit of any rational power of a function is that power of the limit of the function, provided the latter is a real number. it of the sum of two functions is the sum of their limits. We will prove the Sum Rule in Section 1.3. Rules 2-5 are proved in Appendix 2. Rule 6 is proved in more advanced texts, EXAMPLE 1 — Find tim = +4" —3 +5 Solution Starting with the limits lim... x =c and lim,. k =k from Section 1.1, Example 8, and combining them using various parts of Theorem 1, we obtain: 62 Chapter 1: Limits and Continuity Producto Power tim 2? = (im 2) (tim 2) = b) lim (+5) jim ah lim ‘Sum and (2) ©) Tim ats 4 lim xP dct Constant Multiple and (a) 4) lim (4x? — 3) = tim 4x? — lim 3 = 4c? -3 Difference and (c} ©) tin 2° = (tim 22) (tim s) =e seme total oor rove 1) fim (0° + 4r—3) = tim 2° + im x? = 3) sum =eH4e-3 wandie lim (+41 —3) » ee uadent EXAMPLE 2 Find lim, /3x7=3. Solution sin, Vat =3 = VAG Poet alent = 1/2 = Ve-3 =v Qa ‘Two consequences of Theorem | further simplify the task of calculating limits of polynomials and rational funetions. To evaluate the limit of a polynomial function ‘as x approaches c, merely substitute ¢ for x in the formula for the function. To evaluate the limit of a rational function as x approaches a point cat which the denominator is not zero, substitute ¢ for x in the formula for the function. Theorem 2 Limits of Polynomials Can Be Found by Substitution If P(x) = yx" ag 13"! bs tap, then lim P(x) = PCC) = aye” + gsc"! + ++ a, Theorem 3 Limits of Rational Functions Can Be Found by Substitution If the Limit of the Denominator Is Not Zero If P(x) and Q(x) are polynomials and O(c) # 0, then tim 22 _ PO Om ~ Oe) Identifying common factors 1 can be shown that if Q(x) is polynomial and Q(c) = 0, then (x ~ ¢) is a factor of Q(x). Thus, if the numerator and ‘denominator of a rational function of x are both zero at x = ¢, then (x —€) is common factor ® 1.10 The graph of Fix) = (+x 2). (2 —x) in (@) isthe same as the graph of (+2) in (0) except at x= 1, \here f is undefined. The functions have the same limit as x — 1 1.2. Rules for Finding Limits 63 EXAMPLE 3 pee SS ‘This is the limit in Example 1 with ¢ = ~1, now done in one step. a Eliminating Zero Denominators Algebraically ‘Theorem 3 applies only when the denominator of the rational function is not zero a the limit point c. If the denominator is zero, canceling common factors in the ‘numerator and denominator will sometimes reduce the fraction to one whose de- nominator is no longer zero at c. When this happens, we can find the limit by substitution in the simplified fraction, EXAMPLE 4 = Canceling a common factor Evaluate tim, = +*=? Solution We cannot just substitute x =I, because it makes the denominator zero, However, we can factor the numerator and denominator and cancel the common factor to obtain = e+2) a=) Thos See Fig, 110. a EXAMPLE 5 Creating and canceling a common factor VIER Find fim cca Solution We cannot find the limit by substituting h = 0, and the numerator and denominator do not have obvious factors. However, we can create a common factor in the numerator by multiplying it (and the denominator) by the so-called conjugate expression /Z+h + V2, obtained by changing the sign between the square roots: R-v3 _ VUFR- V3 PTR V2 i hb VERVE +h? nV2+h + V2) h W(JTFh + 2) 1 2h + V2 Wie have created a common facor of which we cancel 64 Chapter 1: Limits and Continuity , Oa +h NEFA) Pa..2) 1.11 The limit of the slope of secant PQ 25 = P along the curve is W2v/2) (Example 5). 3 1.12 The graph of fis sandwiched between the graphs of g and h. 1.13 Any function ule) whose graph lies in the region between y = 1+ (x"/2) and ~ (4/8) as limit 1 as x 0. Therefore, lim = 7 The denominators 90 VERO VE longer ou no, L sore can substi Notice thatthe fraction (/T-FT — V/2)/h is the slope of the secant through the point P(2, /2) and the point Q2 +h, V2-+FR) nearby on the curve y= VF. Figure 1.11 shows the secant for ii > 0. Our calculation shows that the limiting value of this slope as Q > P along the curve from either side is 1/(2V3). The Sandwich Theorem ‘The following theorem will enable us to calculate a variety of limits in subsequent chapters. Itis called the Sandwich Theorem because it refers to a function f whose ‘values are sandwiched between the values of two other functions g and hi that have the same limit Z at a point c. Being trapped between the values of two functions that approach Z, the values of f must also approach L (Fig. 1.12). You will find a proof in Appendix 2, Theorem 4 The Sandwich Theorem Suppose that g(x) < f(x) < h(x) forall xin some open interval containing 6 except possibly at x = c itself. Suppose also that tim g(@) = fim h(a) ‘Then lim f(x) = L. EXAMPLE 6 — Given that 1-5 sus) s1+5 forall x £0, find tim (x). Solution Since lim, so (1 —(22/4)) = 1 and lim, (1+ (27/2)) (Fig. 1.13), a the Sandwich Theorem implies that lim, u(x) EXAMPLE 7 — Show that if lime |f(x)| =0, then lim... f(x) =0. Solution Since —|f(x)| < f(x) < |f(x)|, and —| f(x)| and | f(x)| both have limit O.as x approaches c, lim, - f(x) =0 by the Sandwich Theorem. a Exercises 12 65 Exercises 1.2 Limit Calculations Find the limits in Exercises 1-16, 1. fim @x+5) 2 Jim (10—3x) 3. fim (=x? + 5x -2) (8-207 4 4r +8) Stig 90-6 ig. as) 7 yt? 9 sim 10. tim 24? Hm I sy 46 1h fim, 30 — 1 1 im, ce +3)" 13. lim, (5 — yy"? 14, fim Qe 8)" 3 s 15, jim 16. jim 1 tat eres Find the iit in Exercises 17-30 143 18 ln, Teares 20. tig P= BHO Using Limit Rules 31. Suppose lim, .o f(x) =1 and lim, .o g(x) =—S. Name the rules in Theorem that ate used to accomplish steps (a) (D, and (6) ofthe flowing calculation lim 2/0) - gt) i @ lim 2/2) ~ lim g(x) se ecient athe ) (ig 0+) 2 35. 36. 2limy Fes) — fim 6s) Boa (ag 400+ tg 7) ew-c9 7 aaa Let ty) =. lA) = Hs a ens P00) = 2. Name the rules in Theorem I that aze used to accomplish steps (2), (b), nd (c) of the following calculation. SE ny VRC I FONa=roD ~ Ty WOES rOD ° i sme + (b) (is 709) (iy re) (Stim hoy — = ___ @ (im p¢e>) (tim 4 = tim r(x)) _ VO _s ~e=3 Suppose lime fx) = $ and lim, .¢ g(x) = 2. Find 8) tim flee) Dim 2700) fo) © lim (Fe) +3¢0 tim ) Him (f(s) +3¢t2) lim ee Suppose lim. F(X) =O and lime g(x) = ~3. Find 8) tim (@)+3) by tim xf) © tim (ge? ® tim Suppose fim,p f(x) = 7 and fim. g(x) a) tim (70) + 8) by im 62) + als) ©) lim Apts) fim fen/ato Suppose that lim.» p(x) =4,lim,.-2 r(x) = 0, and Jim,.-2 9(0) = —3. Find 8) lim, (p(2) +r(x) +50) 8) im, pO) r(x) 900) im, CAp(e) + Srexy/s00) 66 Chapter 1: Limits and Continuity Limits of Average Rates of Change Because oftheir connection with secant lines, tangents, and instants neous rates, limits of the form. feet hy— flo, i ‘occur frequently in calculus. In Exercises 37-42, evaluate this limit for the given value of x and function f 3. faa, 38 fix)ar x= WB. fi =3x-4, x H. f(x) =n, x= 41. fo) = VE, x=7 fee) Jim xl Using the Sandwich Theorem 43, VSWR = flay = JS for 1 sx 3 find Timo F00) 412 3 < g(x) = 2cosx for all, find limo g(3). 48, a) It can be shown that the inequalities ett hold forall values of x close to zero, What, if anything, does this tell you about 1 Give reasons for your answer GRAPHER Groph yal /),y 1 together for ~2 0, Sp (xsin x)/(2— 20082), and 46. a) pose thatthe inequalities 1 Lecox 1 274 Bsa a) hold for values of x lose to zero. (They do, as you will see in Section 8.10.) Wha, if anything, does this tell you shout L-cosx Lim ? Give reasons for your answer, GRAPHER Graph the equations y = (1/2) — 22/24), (1 cosxy/x?, and y = 1/2 together for -2 1, at what poins ¢ do you automatically know fim. F()? Wha can You say about the value ofthe limit st these points? 48, Suppose that g(x) < f(x) < (x) for all x #2 and suppose that Him, g(x) = tim hx) = 5, Can we conclude anything about the values of fg, and A at x= 22Could £Q) for your answers. 07 Give reasons Y? Could lim,n2 fe 9. 1, find tim £2). F4) and (im, £2? St. 9) i fin 1 10. » nF) 2. acti LP = 4, tif) and ts LP BEsx a) GRAPHER Graph ga) = rsn (1/0) 1 estinte Tim,-.0 (8), Zooming in on the origin as necessary. ) Confirm your estimate in (a) with a proof, GRAPHER Graph h(x) = x cos (1/4) to estimate lim,..9 h(x), 200ming in on the origin as necessary. ) Confirm your estimate in (a) with a proof Target Values and Formal Definitions of Limits In this section we give a formal definition of the limit introduced in the previous two sections. We replace vague phrases like “gets arbitrarily close” in the informal definition with specific conditions that can be applied to any particular example. To do this we first examine how to control the input of a function to ensure that the output is kept within pre t bounds, Keeping Outputs near Target Values ‘We sometimes need to know what input values x will result in output values of the function y .F (2) near a particular target value. How near depends on the context 1.14 Keeping x within 1 unit of x within 2 units of yo = To, 3} by (0.37 Keeping x between 1.75 and 2.28 will keep y between 1.8 and 2.2. will keep y 1.3. Target Values and Formal Definitions of Limits 67 A gas station attendant, asked for $5.00 worth of gas, will try to pump a volume of ‘gas worth $5.00 to the nearest cent. An automobile mechanic grinding a 3.385-in, cylinder will not let the bore exceed this value by more than 0.002 in. A pharmacist ‘making ointments will measure ingredients to the nearest milligram, EXAMPLE 1 Controlling a linear function How close to xy = 4 must we hold the input x to be sure that the output y lies within 2 units of yp = 7? Solution We are asked: For what values of x is |y ~ 7| <2? To find the answer ‘we first express |) ~ 7| in terms of x: ly-71=1@x-)~7| i2x ~ 8 ‘The question then becomes: What values of x satisfy the inequality [2x — 8| < 2? To find out, we solve the inequality 2x —8| <2 -2<2%r-8<2 6<2r<10 Bexe5 -lex-4eL Keeping x within | unit of x9 =4 will keep y within 2 units of yp = 7 (Fig. 1.14). To conc * this Rewie this a Technology Target Values You can experiment with target values on a ‘graphing utility. Graph the function together with a target interval defined by horizontal lines above and below the proposed limit, Adjust the range or use zoom until the function’s behavior inside the target interval is clear, Then ‘observe what happens when you try to find an interval of x-values that will keep the function values within the target interval. (See also Exercises 7-14 and CAS Exercises 61-64.) For example, try this for f(x) = V3x—2 and the target interval (1.8, 2.2) ‘on the y-axis, That is, graph yy = f(x) and the lines y2 = 1.8, ys = 2.2. Then try the target intervals (1.98, 2.02) and (1.9998, 2.0002). 68 Chapter 1: Limits and Continuity Stipes shout Tm vide Seine 3 Liquid volume VE buh cc 1.15 A 1-L measuring cup (a), modeled as. 2 Fight circular cylinder (b) of radius 1 =6 cm (Example 2) 1.16 A preliminary stage in the development of the definition of limit. EXAMPLE 2 Why the stripes on a 1-liter kitchen measuring cup are about a millimeter wide ‘The interior of a typical 1-L measuring cup is a right circular cylinder of radius 6 em (Fig. 1.15). The volume of water we put in the cup is therefore a function of the level A to which the cup is filled, the formula being V = n6°A = 360h. How closely must we measure A to measure out I L. of water (1000 em®) with an error of no more than 1% (10 em?)? Solution We want to know in what interval to hold values of h to make V satisfy the inequality V ~ 1000] = 36h 1000} < 10. To find out, we solve the inequality: [36x — 1000) < 10 10 < 36h — 1000 < 10 990 < 36h < 1010 990, < 1010 36r ~" ~ 36 88 Sh=89 / \ rounded vp, routded down, tobe safe tobe safe ‘The interval in which we should hold iis about 8.9 ~ 8.8 = 0.1 em wide (1 mm), With stripes 1 mm wide, we ean expect to measure a liter of water with an accuracy of 1%, which is more than enough accuracy for cooking a The Precise Definition of Limit In atarget-value problem, we determine how close to hold a variable x to a particular value xp to ensure that the outputs f(x) of some function lie within a prescribed interval about a target value L, To show that the limit of f() as x — x9 actually ‘equals L, we must be able to show that the gap between f(x) and L can be made less than any prescribed error, no matter how small, by holding x close enough to Suppose we are watching the values of a function f(x) as x approaches x) (without taking on the value of xy itself). Certainly we want to be able to say that F(x) stays within one-tenth of a unit of Las soon as x stays within some distance 4 of xo (Fig. 1.16). But that in itself is not enough, because as x continues on its course toward xp, what is to prevent f(x) from jitering about within the interval from L ~ 1/10 to L + 1/10 without tending toward L? We can be told that the error can be no more than 1/100 or 1/1000 or 1/100,000. Each time, we find a new 5-interval about xy so that keeping x within that interval satisfies the new error tolerance, And each time the possibility exists that (x) jitters away from L at the last minute. ‘The following figures illustrate the problem. You can think of ths as a quarrel between a skeptic and a scholar. The skeptic presents e-challenges to prove that 41.3. Target Values and Formal Definitions of Limits 69 the limit does not exist or, more precisely, that there is room for doubt, and the scholar answers every challenge with a 3-interval around x. 408 8ym9 "F080 ‘Thechallenge Response: [New challenge: : Makes) — Ll 0, there exists a corresponding number 6 > 0 such that for all x 0<|r-ml <3 => Ifoy—Li 0 we have to find a suitable 5 > Oso that if x #1 and xis within distance 8 of xo = 1, that is, if OK 0 be given, We must find 6 > 0 such that for all x O<|x—xol <5 — implies [x—xol 0 be given, We must find 5 > 0 such that forall x O<|x-xol <8 implies k-AI 0 that works for € = 1. That is, find a 5 > 0 such that for all x O<|x—5|<8 => We-1-2)<1 72 Chapter 1: Limits and Continuity 3 3 tee pl 2 e —10 1.21 An open interval of radius 3 about X¢= 5 will lie inside the open interval @, 10). 1.22 The function and intervals in Example 5, Solution We organize the search into two steps. First we solve the inequality I= 1 = 2] <1'to find an interval (a, b) about xo = 5 on which the inequality holds for all x # xp. Then we find a value of 5 > 0 that places the interval $— 8 < x < 5-46 (centered at xy = 5) inside the interval (a, 6). Step 1: Solve the inequality |x/x—1 —2| < 1 to find an interval about xy = 5 on which the inequality holds for all x # x WeeT=21 <1 -l 0 that places the centered interval S—§ 0 Algebraically ‘The process of finding a 8 > 0 such that for all x O 0 that places the open interval (xo ~ 8, x0 +8) centered at xp inside the interval (a, b). The inequality | (x) — LI < € will hold for all x 7 xo in this 8-interval EXAMPLE 6 Prove that lim,.2 f(x) =4 if Solution Our task is to show that given € > O there exists a § > 0 such that for all x O [fay-dice Step 1: Solve the inequality | f(x) ~ 4| < € 10 find an open interval about don which the inequality holds for all x # xy oz aN Ify-aice is completes the proof. Why was it allright to assume € < 4? Because, in finding a 8 such that for all x, 0.< |x ~ 2) <5 implied | f(x) ~ 4] < € <4, we found a 6 that would work for any larger € as well, Finally, notice the freedom we gained in letting §=min (2~/T=<, V4+e — 2}. We did not have to spend time deciding which, if either, number was the smaller of the two, We just let § represent the smaller and went on to finish the argument a Using the Definition to Prove Theorems We do not usually rely on the formal definition of limit to verify specific limits such as those in the preceding examples. Rather we appeal to general theorems about limits, in particular the theorems of Section 1.2. The definition is used to prove these theorems. As an example, we prove part 1 of Theorem I, the Sum Rule. EXAMPLE 7 Proving the rule for the limit of a sum Given that lim. f(x) = L and Tim... g(x) = M, prove that fim (f() + ¢@)) = L+M. Solution Let € > 0 be given. We want to find a positive number 8 such that for all x O [f+ aGQ)~(L+ MII 0 such that for all x O Osuch that forall x O<-el [glx)-M) O such that forall x, 0 < lena) 0 such that for ll x Oek-mi 0. In each case, ind an open interval about xo on which the inequality | f(x) — L| < € holds. Then give a value for 3 > O such that for all x satisfying 0 < |x ~xo| <6 the inequality | f(x) ~L| < « tos. 1S fa)axtl, L=5, w= a ver, Vi L=1/2, % VIO=, L=3, a9 =4, w=, Ws, L= 1/4, m4, «005 = 001 «= 002 ou Ix, L=-l w= -1, = 01 -5, ball, w=4, c= 120/x, me m=O, L mx, m>0, mx +b, m>0, L=(m/2)+b, m= 1/2 >0 30. f(x) = med, m>0, L=mbb, x6 More on Formal Limits Bach of Exercises 31-36 gives a function f(x), @ point x9, and a positive number ¢, Find Z FG). Then find a number 5 > 0 Exercises 13. 75 sch that forall x O [fa)-Lice Bf) = 3-28, y=3, € = 002 35. fle) = ITB, w= 8 36 fix) =4/x, x = 2, Prove the limit statements in Exercises 37-80. 3 lim =) =5 38 im x7) Blin F=3=2 40. ly VIB =2 1 tim f60) = ear 42 in, fe) = 47. tim foy=2 it fone [FO TET 48. tim far=o it fon= [Ze T55 (Genoa Maberatea) 16 {ene Maman) Chapter 1: Limits and Continuity Theory and Examples 5h. 82, 33. ss, Define what it means to say that fo Deine what it means to say that lim, g(x A wrong statement about limits. Show by example that the following statement is wrong. ‘The number L is the limit of f(x) as x approaches xp if fC) gets closer to L as x approaches xo, Explain why the function in your example does not have the given value of Las @ limit as x > x0 Another wrong statement about limits. Show by example thatthe following statement is wrong, The number L is the limit of f(x) as x approaches 4 if, given any € > 0, there exists a value of x for which | f(x) ~L| < Explain why the function in your example does not have the given value of Z asa limit as ¥ 2, Grinding engine cylinders. Before contracting to grind engine cylinders to a cross-section area of 9 im’, you need to know how ‘much deviation from the ideal eylindee diameter of xp = 3.385 in. you ean allow and still have the area come within 0.01 ia? of the required 9 in’, To find out, you let A = 27(x/2)* and look for the interval in which you must hold x to make |4 ~ 9] < 0.01 ‘What interval do you find? ‘Manufacturing electrical resistors. Ohm's law for electrical circuits like the one shown in Fig. 1.24 states that V = RY. In this equation, Visa constant voltage, is the curent in amperes, and R is the resistance in ohms. Your firm has been asked to supply the resistors for a circuit in which V will be 120 volts and isto be 50.1 amp. In what interval does R have to to be within 0.1 amp of the target value fy = 5? for! 1.24 The circuit in Exercise $6, When Is a Number L Not the Limit of f(x) as x x0? We can prove that lim. f(3) # L by providing an € > such that no possible § > 0 satisfies the condition For all x, lra@y-L <6 ‘We accomplish this for our candidate « by showing that for each > O there exists a value of x such that O and O< [rs <3 lray- Lee RFP wre vale of for which O< [a ~ x < Band fa) — | se x orel xth xe y Exercises 13. 77 a) Let = 1/2. Show that no possible 6 > 0 satisfies the fol- 6. a) Forte function graphed here, show that lim. g(x) # 2 Towing condition: bb) Does lim, . g(x) appear to exist? Ifso, what is the value Forallx, O<|x-N | f@)-2) < 1/2 ‘of the limit? If not, why not? ‘That is, for each 5 > 0 show that there is a value of x sueh that O 1/2. This will show that lisa: f() #2. b) Show that tim...) £0) #1. ©) Show that tim, sy f(x) #15. Bo x22 Let AG) x=2 2 x>2, © CAS Explorations and Projects In Exercises 61-66, you wil further explore finding deltas graphically Use a CAS to perform the following steps: 2) Plot the function y = f(x) near the point being approached. ) Guess the value of the limit Z and then evaluate the limit sym- bolicaly to see if you guessed correctly. ‘¢)_ Using the value ¢ = 0.2, graph the banding lines yy = y= Le together withthe function f near xy, 14), From your graph in part (©), estimate a 5 > 0 such that fr all x «and Show thet O<|s-ml |fa)-Li —_— | Postve side ofa —+—_«—__;— Extensions of the Limit Concept In this section we extend the concept of limit to 1. one-sided limits, which are limits as x approaches a from the left-hand side or the right-hand side only, 2. infinite limits, which are not really limits at all, but provide useful symbols and language for describing the behavior of functions whose values become arbitrarily large, positive or negative. One-Sided Limits ‘To have a limit L asx approaches a, a function f must be defined on Doth sides of 4, and its values. (4) must approach L as x approaches a from either side, Because of this, ordinary limits are sometimes called two-sided limits. Itis possible for a function to approach limiting value as + approaches a from only one sid, ether from the right or from the left. In this ease we say that f has a one-sided (either right-hand or left-hand) limit at a. The function f(x) = x/|x\ graphed in Fig. 1.25 has limit 1 asx approaches zero from the right, and limit —1 438 x approaches zero from the left Definition Informal Definition of Right-hand and Left-hand Limits Let f(x) be defined on an interval (a,b) where a 0°, the values of f grow without bound, eventually reaching and surpassing every positive real number, That is, given any positive real number B, however large, the values of f become larger still (Fig. 1.29). Thus, fhas no limit as x — O*. Itis nevertheless convenient to describe the behavior of f by saying that f(x) approaches 00 as. x > 0*. We write 1 lim f(x) = fim, © = oo. In writing this, we are not saying that the limit exists. Nor are we saying that there is a real number 00, for there is no such number. Rather, we are saying that lim, +o (1/x) does not exist because 1/x becomes arbitrarily large and positive as x > 0", Asx + 07, the values of f(x) = I/x become arbitrarily large and negative. Given any negative real number —B, the values of f eventually lie below —B. (See Fig. 1.29.) We write 1 lim f(x) = im, 00. Again, we are not saying that the limit exists and equals the number —oc. There is no real number —o0. We are describing the behavior of a function whose limit as x + 0" does not exist because its values become arbitrarily large and negative. 1.30 Near x =1, the function y= ~ 1) behaves the way the function y= tx behaves near x = 0. its graph is the graph of y = Wx shifted 1 Unit to the right. ‘No matter how igh the eaph pees eter 1.31 The graphs of the functions in Example 5. 1.4. Extensions of the Limit Concept 81 EXAMPLE 4 One-sided infinite limits Find tim, <1 and tim Geometric Solution The graph of y = 1/(x — 1) is the graph of y = 1/x shifted unit to the right (Fig. 1.30). Therefore, y = 1/(¢ ~ 1) behaves near I exactly the way y = 1/x behaves near 0: 1 Jim, co and im Analytic Solution ‘Think about the number x ~ 1 and its reciprocal, As x > 1*, we have (x — 1) — 0* and 1/(x — 1) + 00. As x = 1°, we have (x — 1) > 0 and 1/( = 1) + 00, Q EXAMPLE 5 Two-sided infinite limits Discuss the behavior of a) sey = 4 nears b) gx) = ) B= 7 Solution a) As x approaches zero from either side, the values of 1/x* are positive and become arbitrarily large (Fig. 1.319) , ‘nm 2 ions b) The graph of g(x) = 1/(x +3) is the graph of f(x) = 1/x? shifted 3 units to te it gly Tee, g eves ear 3 enacly te ay behaves wear i =i zu B90) = Ne, ae a Te fnson y= 1/s shows no coset Behavior a8 x» 0. We have I/x — ooifx > 0*, but 1/x > —ooifx — 0”. All wecan say about lim, (1/x) is that it does not exist. The function y = 1/x? is different. Its values approach in- finity as x approaches zero from either side, so we can say that lim,..9 (1/x7) = 00. EXAMPLE 6 Rational functions can behave in various ways near zeros of their denominators. (27 pores! » =m a2 x2 1 b) lim = tim 7? stim > eh @-DetD eyed x3 oO lim = tim 4-3 = co The sls te neative “3 OB @-De+D ory > Bancar 82 Chapter 1: Limits and Continuity 0) jp» $+ 1.32 Diagram for the definition of right-hand limit. tet ' ee ee] : 8 % 1.33 Diagram for the definition of leftchand limit = ‘The yale re postive 9 @-Da45 fered smer? 1-3 ©) tim 53 = tim TF doesnot exit, See) ant oh ) tim 2—* 2) oe st at @-F ea In parts (a) and (b) the effect ofthe zero inthe denominator atx = 2is canceled because the numerator is zero there also, Thus finite limit exists. This is not true in part (f), where cancellation sil leaves a zero in the denominator. a Precise Definitions of One-sided Limits ‘The formal definition of two-sided limit in Section 1.3 is readily modified for one-sided limits Definitions Right-hand Limit We say that f(x) has right-hand limit Z at xo, and write lim fy =L (See Fig. 1.32) if for every number € > 0 there exists a corresponding number > 0 such that for all x Xo Left-hand Limit We say that fhas left-hand limit L at xo, and write lim f(x) (See Fig. 1.33) Ife) -Li 0 there exists a comesponding number 8 > 0 such that for all x woS [f@)-Li If@)-Ll 0 such that for all x O f>B. 2. We say that f(x) approaches minus infinity as x approaches x9, and write 20, Jim fe) if for every negative real number ~B there exists a corresponding 5 > 0 such that for all x OK fae-B ‘The precise definitions of one-sided infinite limits at xp are similar and are slated in the exercises, Exercises 1.4 Finding Limits Graphically 1. Which of the following statements about the funetion y= f(x) a) ‘graphed here are rue, and which are false? ») fim soy = 8 4) fim f00)= him foo) ©) fim, flay exiss © lim soy =0 ®) tim fo) =1 by tim foo =1 tim fey JD fim f(xy =2 1) fim f(a) does not exist. 1) im f(a) = 0 84 Chapter 1: Limits and Continuity 2, Which of the following statements about the function y = fx) Db) Does lim.s2 (x) exist? If so, what i it? I'mot, why not? graphed here are true, and which are false? ©) Find fim, f(0) and tim, y+ f). 4) Doestimy 1 f(x) exist? 1FS0, whats i? IFnot, why not? 0, x<0 SIO =) gl goo, a) im f(s) © lim sey = 1) tim cx) doesnot exist. @) im soy =2 jim fey =1 8) lim so) = fim soo) {£(2) exists at every cin the open interval (—1, 1). £) lim f(x) does noteist 4) lim f(2) exists atevery cin the open interval (1,3) ky lim /(4) does notexist, cd a) ») ° a) Find lim, or (8) and lim,s- f(0) 1b) Does lim, .2 f(x) exist? If 50, what is it? Tf not, why not? ©) Find lim, G2) and lime f(0) 4) Does im, 4 (2) exist? 150, what is iW If not, why not? bon x2 ara so= |? Ere a) b) ° ») a) Find lim,.ar fG) lime- f(x), and £2) ° Does lim. .o* (2) exist? If 0, what is it? IF not, why not? Does lim, .y- f(x) exist? If s0, what is ic? IF not, why no? Does tim, 0 /(x) exist? If so, what is it? If not, why not? isin (Us) (cones y Mabe Does tim, s+ (x) exist? If so, what i it? If not, why not? Does limo (x) exist? Iso, what is it? If not, why not? Does fimo g(2) exist? If so, what i it? If not, why not? a fe. r#l Graph f¢9) {e ae Find imy- f(a) and tim, >. Does timy-»1 Ja) exist? 1°50, what is it? If not, why not? 8a) by Find tim, y+ G2) and tim..1- f(2. ©) Does tims f(x) exist? IF 0, what isi? IF not, why not? Graph the functions in Exercises 9 and 10, Then answer these ques- a) What are the domain and range of f? by At what points cif any, does Tim, .. (x) exist? ©) At what points does only the left-hand limit exist? 4) At what points does only the right-hand limit exist? vI-¥ if Osred 9% fa)=41 if 1ex<2 2 if x=2 x if -Isr<0, f= fi if x=0 0 if re- o or O 0? Find the limits in Exercises 43-46. 43 im a) by sor 86 Chapter 1 mits and Continuity stm (9) a) 10 b 10 a) x0 boro 9 xslt a xsi Theory and Examples 47. Once you know Him.» f(8) and Tim,..,-_ f(a) at an interior point ofthe domain off, do you then know lit. (2)? Give Feasons for your answer 48. If you know that lim,... f(x) exists, can you find its value by calculating lim...» (0)? Give reasons for your answer. 49. Suppose that f is an odd function of x. Does knowing that Tim, f(x) = 3 tell you anything about lim, o- f(2)? Give reasons for your answer. ‘$0. Suppose that fis an even function of x: Does knowing that Tim.» f(x) = 7 tell you anything about either lia... f(a) or Tim,..-2+ f(4)? Give reasons for your answer Formal Definitions of One-sided Limits SI. Given ¢ > 0, find an interval f = (5,548), > 0, such that if lies in, then x—5 <€, What limit is being verified and ‘what is its value? ‘52, Given € > 0, find an interval 1 = (4 ~ 4,4), > 0, such that if “lies in, then =x < €, What limit is being verified and ‘what is its value? Us the defo of sighed an td Kins 6 ove te Wn tre aces 5 and 3 8 tin Fant 54, im 1 BB emai > $5. Find (a) lim, y> [and (b) lim, ago” [xfs then use limit def initions to verify your findings. (¢) Based on your conclusions in (@) and (b), can anything be said about lim, .4qp [x]? Give reasons for your answers. x<0 x>0. 1 F(8) and (b) Him, o> (25 then use limit 0 such that forall x wexB Modify the definition to cover the following cases. a) fim f() = 00 b) fim fo = ©) lim f(x) = -00 Use the formal definitions from Exercise 61 to prove the limit state ‘ments in Exercises 62-67. 63. im 65. lim 67. tien 1.5 Continuity 87 Continuity When we plot function values generated in the laboratory or collected in the fel, we offen connect the plotted points with an unbroken eurve to show what the function's values are likely to have been atthe times we did not measure. In doing s0, we are assuming that we are working with a continuous function, a function ‘whose outputs vary continuously with the inputs and do not jump from one value {0 another without taking on the values in between, So many physical processes proceed continuously that throughout the eigh- teenth and nineteenth centuries it rarely occurred fo anyone to look for any other kind of behavior. It came as quite a surprise when the physicists of the 1920s discovered thatthe vibrating atoms in a hydrogen molecule can oscillate only at discrete energy levels, that light comes in particles, and that, when heated, atoms emit light at discrete frequencies and not in continuous spectra. As a result of these and other discoveries, and because of the heavy use of discrete functions in com- puter science and statistics, the issue of continuity has become one of practical as, well as theoretical importance In this section, we define continuity, show how to tell whether a function is continuous at a given point, and examine the intermediate value property of continuous functions. Continuity at a Point In practice, most functions of a real variable have domains that are intervals or unions of separate intervals, and it is natural to restrict our study of continuity to functions with these domains. This leaves us with only three kinds of poins to consider: interior points (points that lie in an open interval in the domain), left endpoints, and right endpoints. Definition A function f is continuous at an interior point x = c of its domain if lim f@) = fe) In Fig. 1.36 on the following page, the first funetion is continuous at x = 0. ‘The function in (b) would be continuous if it had f(0) = 1. The function in (c) ‘would be continuous if f(0) were 1 instead of 2. The discontinuities in (b) and (©) are removable. Each funetion has a limit as x > 0, and we can remove the discontinuity by setting f(0) equal to this limit. The discontinuities in parts (@)-(i) of Fig. 1.36 are more serious: lim,.o f(x) does not exist and there is no way to improve the situation by changing fat 0. ‘The step function in (d) has a jump discontinuity: the one-sided limits exist but have different values. The function f(x) = 1/x? in (e) has an infinite discontinuity. Jumps and infinite discontinuities are the ones most frequently encountered, but there are others. The function in (F) is discontinuous at the origin because it oscillates too much to have a limit as x > 0. 88 Chapter 1: Limits and Continuity | (y= 0) | 1.36. The function in (a) is continuous at X= 0; the functions in (b)-(f) are not a) y: = int x incorrectly graphed in connected mode. b) y; = int x correctly graphed in dot mode. (y= fo) yas) © ® © Technology Deceptive Pictures A graphing utility (calculator or Computer Algebra System—CAS*) plots a graph much as you do when plotting by hand: by plotting points, or pixels, and then connecting them in succession. The resulting picture may be misleading when points on opposite sides of @ point of discontinuity in the graph are incorrectly connected. To avoid incorrect connections some systems allow you to use a “dot mode.” which plots only the points. Dot mode, however, may not reveal enough information to portray the {rue behavior of the graph, Try the following four functions on your graphing device. If you can, plot them in both “connected” and “dot” modes. atx=2 jump discontinuity atx=0 oscillating discontinuity atx=2 infinite discontinuity removable discontinuity “Rhymes with lass. Continuity Twosided fiom the right c9ntimuity— Continuity aS yas) oe ed, 2 € > 1.37 Continuity at points a, 6, and c. x 2 0 2 1.38. Continuous at every domain point. 1.39 Right-continuous at the origin. 1.40 This function, defined on the closed interval [0,4 is discontinuous at x = 1, 2, and 4. Its continuous at all other points of its domain, 1.5 Continuity 89 Continuity at endpoints is defined by taking one-sided limits. Definition A function f is continuous at a left endpoint a of its domain if dim, ft) = fl@) and continuous at a right endpoint x = b of its domain if Jim Fe) = F. In general, a function f is right-continuous (continuous from the right) at a point x = cin itsdomainiflim, ,.+ f(x) (c). Itis left-continuous (continuous: from the left) at ¢ if lim,s.- f(x) = f(e). Thus, @ function is continuous at a left endpoint a of its domain if itis right-continuous at a and continuous ata right endpoint b of its domain if it is left-continuous at b. A function is continuous at an interior point ¢ of its domain if and only if itis both right-continuous and left-continuous at c (Fig. 1.37). EXAMPLE 1 The function f(x) = Y—¥" is continuous at every point of its domain, (=2,2] (Fig, 1.38). This includes x = —2, where f is right-continuous, and x =2, where f is left-continuous. a EXAMPLE 2 The unit step function U(x), graphed in Fig. 1.39, is sgl continuous at x = 0, but is neither left-continuous nor continuous there. ‘We summarize continuity at a point in the form of a test. Continuity Test A function f(x) is continuous at = if and only iit meets the following three conditions 1. f@ exis 2 Time f(x) exists 3. lime FO) = FO (€ lies in the domain of f) Cf has a limit as x > ©) (the timit equals the function value) For one-sided continuity and continuity at an endpoint, the limits in parts 2 and 3 of the test should be replaced by the appropriate one-sided limits. EXAMPLE 3 Consider the function y = f(x) in Fig. 1.40, whose domain is the closed interval (0, 4]. Discuss the continuity of f at x = 0,1,2,3, and 4 90 Chapter 1: Limits and Continuity Solution The continuity test gives the following results: 8) f is continuous at x =0 because F(0) exists (FO) = 1), fi) Lim, o> f(x) = 1 (the right-hand limit exists at this left endpoint), iit) tim, sor f() = f(O) (the limit equals the function value), b) _f is discontinuous at x = I because lim, .. f(x) does not exist. Part 2 of the test fails: f has different right and left-hand limits atthe interior point x = 1 However, fis right-continuous at x = | because i) FU) exists (FO) =D. ) tim, fx) = 1 (the right-hand limit exists at x = 1), fil) Lim... f(x) = FCA) the right-hand limit equals the function value). ©) f is discontinuous at x = 2 because lim, > f(x) # f(2). Part 3 of the test hails. 4) f is continuous at x = 3 because 1) £Q) exists (/3) =2), i) Him,.3 f() = 2 (the limit exists at x = 2), iil) Tim, Ce) = f@) the limit equals the function value). ©) fis discontinuous at the right endpoint x = 4 because lim. f(x) # (4). ‘The right-endpoint version of Part 3 of the test fails, Q Rules of Continuity Ie follows from Theorem 1 in Section 1.2 that if two functions are continuous at 4 point, then various algebraic combinations of those functions are continuous at that point Theorem 6 Continuity of Algebraic Combinations If functions f and g are continuous at x = c, then the following functions are continuous at x = ¢: L ftgandf—g fs kf, where k is any number Fg (provided g(c) # 0) (£00)! (provided f(x))"" is defined on an interval containing c, and m and n. are integers) As a consequence, polynomials and rational functions are continuous at every point where they are defined, 1.41 The sharp comer does not prevent the function from being continuous at the origin (Example 5). 1LA2 The continuity of composites. 1.5 Continuity 91 Theorem 7 Continuity of Polynomials and Rational Functions Every polynomial is continuous at every point ofthe real line, Every rational function is continuous at every point where its denominator is different from ero. EXAMPLE 4 The functions f(x) x4 + 20 and g(x) = Sx(x — 2) are contin- uous at every value of x. The function fla) __x*+20 re) = L420 >= sa) ~ Sxe—2) is continuous at every value of x except x = 0 and x = 2, where the denominator is 0. a EXAMPLES Continuity of f(x) = |x ‘The function f(x) = [x] is continuous at every value of x (Fig. 1.41). If.x > 0, we have f(x) = x, a polynomial. If x <0, we have f(x) = —x, another polynomial Finally, at the origin, limy..g [| = 0 = [0 a EXAMPLE 6 Continuity of trigonometric functions We will show in Chapter 2 that the functions sinx and cos.x are continuous at every value of x. Accordingly, the quotients sinx cosx tnx = 2% goer = SSE 1 1 are continuous at every point where they are defined, a Theorem & tells us that continuity is preserved under the operation of compo- sition, Theorem 8 Continuity of Composites If f is continuous atc, and g is continuous at f(c), then g o f is continuous at ¢ (see Fig. 1.42) Comin aan Continuous —~ aso) ¢ a) sified 92 Chapter 1: Limits and Continuity ® \d a3 aoe > a T einer eee teem eee op 1.43 (a) The graph of 24x-6 10) = 5 and (b) the graph of its continuous extension x43 Fu = 225 (Example 8). ‘The continuity of composites holds for any finite number of functions. The only requirement is that each function be continuous where it is applied. For an outline of the proof of Theorem 8, see Exercise 6 in Appendix 2. EXAMPLE 7 The following functions are continuous everywhere on their re- spective domains a) yave Theowems 6 and 7 (ational pone of a polymi 7 5 Theos and 7 (a pls Theres Tat ¥ (er Db) y= VF Be—S ofp or compotion witht guar) cost” Theorems 6,7 and power. enpenie pret. poy nia 9 Tea and quotes) -2 Teorey 7 and 8 (eompenite of abyolute value anda rational 9 y= function a Continuous Extension to a Point As we saw in Section 1.2, a rational function may have a limit even at a point where its denominator is zero. If f(c) is not defined, but lim, f(x) = L exists, we can define anew function F(x) by the rule roe { fo e is in the domain of f ‘The function F is continuous at x = c. It is called the continuous extension of {f to x =¢, For rational functions J, continuous extensions are usually found by ‘canceling common factors EXAMPLE 8 Show that P+x-6 JO =F has a continuous extension to x = 2, and find that extension, Solution Although (2) is not defined, if x #2 we have (= 2043) +3 @-DatD 342 ‘The function x43 Fa = is equal to f(x) for x # 2, but is also continuous at x = 2, having there the value of 5/4, Thus F is the continuous extension of f to x = 2, and Pex ‘mead ‘The graph of f is shown in Fig. 1.43. The continuous extension F has the same ‘graph except with no hole at (2, 5/4). a Continuity on Intervals A function is called continuous if it is continuous everywhere in its domain. A. function that is not continuous throughout its entire domain may still be continuous when restricted to particular intervals within the domain 1.5 Continuity 93 A function f is said to be continuous on an interval fin its domain if lim, sc f(%) = f(6) at every interior point ¢ and if the appropriate one-sided limits equal the function values at any endpoints / may contain, A function continuous on ‘an interval [is automatically continuous on any interval contained in 1. Polynomials ‘are continuous on every interval, and rational functions are continuous on every interval on which they are defined. EXAMPLE 9 Functions continuous on intervals (2) Continuo () Continvous om (-e2, 0) and (0, <2) 7 —— or (Continuous on (0,0) nd 10,2) fd) Continuous on (0, <9) a Functions that are continuous on intervals have properties that make them partic- 7.44, The function Being continuous on larly useful in mathematics and is applications. One ofthese isthe intermediate tnd f(b), ad value property. A function is said to have the intermediate value property if it never takes on two values without taking on all the values in between. Theorem 9 The Intermediate Value Theorem Suppose f(x) is continuous on an interval I, and a and b are any to points of I. Then if yp is a number between f(a) and f(b), there exists a number ¢ between a and b such that f(c) = Ju (Fig. 14), ‘The proof of the Intermediate Value Theorem depends on the completeness property of the real number system and can be found in more advanced texts ose The continuity of f on J is essential to the theorem. If J is discontinuous at does not take on any value between even one point of /, the theotem's conclusion may fail, as it does for the function f(0) = 0 and f(1) = 1 ‘graphed in Fig. 145. 1.45 The function Fl) 94 Chapter 1: Limits and Continuity 1.46 A graphical solution of the equation x° — x We graph the function fbx) = 9° —x— 1 and, with successive screen enlargements, estimate the coordinates of the point where the ‘graph crosses the x-axis. A Consequence for Graphing: Connectivity Theorem 9 is the reason the graph of a function continuous on an interval 1 cannot have any breaks. It will be connected, a single, unbroken curve, like the graph of sin.x. It will not have jumps like the graph of the greatest integer function |x) or separate branches like the graph of 1/sx. ‘The Consequence for Root Finding We call a solution of the equation f(x) 01a root or zero of the function f. The Intermediate Value Theorem tells us that if f is continuous, then any interval on which f changes sign must contain a zer0 of the function This observation i the basis of the way we solve equations of the form f(x) = 0 with a graphing calculator or computer grapher (when f is continuous). The solutions are the x-intecepts of the graph of f. We graph the function y = f(x) over a large interval to see roughly where its zeros are. Then we zoom in on the iersection points one ata time to estimate their coordinates. Figure 1.46 shows a typical sequence of steps in a graphical solution of the equation x* — x ~ 1 =. Graphical procedures for solving equations and finding zeros of functions, while instructive, are relatively slow. We usually get fester results ftom numerical methods, as you will see in Section 3.8. a gi aeeea ae Hy First we make a graph with a relatively We change the viewing window to ‘Wechange the window to lace sae. It eveals a root (ra) lsns2-l2y=l, Wenow sce Isr 135,-01=y=01 between = land = 2 thatthe roo les Between 3 and The sot los betweun 1.32 and 1.33 LA A L 133/135 Tana Taasni7 128719} We chinge the window to We change the window wo Aero more enlargements, we arive 132s¥= 13,001 sy =001 The rot lies betwen 1324 and 1.525. 13M Se = 1.325, -0.001 ata sereen that shows the foot tobe ‘The rot lies between 1.3247 and 1.3248, ppeoximataly 324718 Exercises 1.5 95 EXAMPLE 10 Is any real umber exactly | less than its cube? Solution This is the question that gave rise to the equation we just solved. Any stich number must satisfy the equation x = x? — 1 or x —x —1=0. Hence, we are looking for a zero of f(x) =x? — x — 1. By trial, we find that f(1) = —1 and £(2) =5 and conclude from Theorem 9 that there is at least one number in [1, 2] where f is zero. So, yes, there is a number that is I less than its cube, and we just estimated its value graphically to be about 1.3247 18. a Exercises 1.5 Continuity from Graphs In Exercises 1-4, say whether the function graphed is continuous on [-1, 3]. If not, where does it fail to be continuous and why? 1 2 =) tA; 1 1 x ~ 111. < 1.47 The graph for Exercises 5-1 0 , 2 3 a) . 2 3 ‘The grapt ® 5-10. a 4 7. a) Is f defined at x = 22 (Look at the definition of f.) t ; b) Is f continuous at x = 2? ym hee) y= ke) ‘8. At what values of x is f continuous? ar 9, What value should be assigned to f(2) to make the extended func- FS tion continuous at x = 2? ; on an 10. To whit new value should (1) be changed to remove the dis- or) re meses continuity? about the function Applying the Continuity Test sme tee {At ich points do the functions inthe fllowing exercises fi to be 2 Onze! continuous? At which points, i any, are the discontinuities removable? sev=y i, : sot removable? Give reasons for your answers W244, Lex? 11 Exercise 1, Section 1412. Exerise2, Section 14 @ 2eres At what points are the functions in Exercises 13-28 continuous? ‘graphed in Fig. 1.47. m 5 a) Does f(=1) exist? y= -e Moaoit 1b) Does fim.» exist? Ca my ©) Does Him. fx) = f(D? 15. y _ 6 y= 8 @) Is f continuous at x = —17 eee pene 6 a) Does f (1) exist? 17. y= [x= A +sinx 1 y= DY Does fiat f) exist? +12 ©) Dees lim ft8) =F)? a yoEt d) Is f continuous at x = 1? ue x 2 cose 96 Chapter 1: Limits and Continuity ese de 22 yt Vea “+ sin'x 24. vars 26, y= R=1 2. y= (Qe - 1)? Be y=Q-x's Limits of Composite Functions Fin th fimits in Exercises 29-34 B. in since ~sinz 20. ty sn (Ect) 31, tim see (y sec? y — tan? y — 1) 52. fing tn (F con ins!) 33. cos (A) n° | p= Seem lim, Veneta 4 5S 34 in, ect + 35am Continuous Extensions 135. Define @(3) in a way that extends g(x) = (x2 — 9)/(x — 3) to be continuous at x = 3 36, Define h(2) in a way that extends h(t) = (#2 + 3¢ — 10)/(¢ ~2) fo be continuous at ¢ = 2. 31. Define f(1) in a way that extends f(s) = (¢* — 1)/(s? 1) 10 be continuous at s = 1 38, Define g(4)ina way that extends g(x) = (x tobe continuous atx = 4 39, For what value of a is so= [Ee TS 16 /oe ae) continuous at every x? 40, For what value of b is continuous at every x? 8% Grapher Explorations—Continuous Extension In Bxeoises 41-44, graph the function f to see whether it appears to have a continuous extension to the origi. If it does, use TRACE and ZOOM to find a good candidate forthe extended functions value at {x= 0.1 he function does not appear to havea continuous extension, can it be extended to be continuous atthe origin from the right or fromthe lef? 130, what do you think the extended functions vale(s) should be? 10* 10" 4 fo) = 2. fy= 8 10 = TF 44 fey = 120) Theory and Examples 45. A continuous function y = f(x) is known to be negative atx = 0 ‘and positive at x= 1. Why does the equation f(x) = 0 have at teat one solution between x =O and x= 1? Mustate with a sketch 46, Explain why the equation cos.x =x has atleast one solution. 47, Show that the equation x? — 15x + 1 = 0 has three solutions in the interval [—4, 4]. ‘Show that the function F(x) = (x —a)*(x — b)? +.x takes on the vale (a +B)? for some vale of 49. If f(x) = x° ~ 8x + 10, show that there are values ¢ for which Fl@) equa (a) 2; ) V3 (6) $00,000 50, Explain why the following five statements ask forthe same in- formation a) Find the roots of f(x) =x? — 3x —1, 1) Findihes-coordnates ofthe pons where the ere y = erence te fine y = 3 ©) Find all the values of x for which x? ~ 3x {Find the coordinates ofthe points wher the cube carve yaa? 3x ersses the line Y= 1 © Solve the equation 3x10. 51, Give am example of fmcton (2) tha is continous for al vals of «except x = 2, where thas a removable discontny Explain bow you know tat discontnoos stx =, and Row you know the discontinuity s removable. 52. Give an example of a function g(x) that is continuous for all values of x except x = —I, where it has a nonremovable diseon- tinuity. Explain how you know that g is discontinuous there and ‘why the discontinuity is not removable, #53, + A function discontinuous at every point 8) Use the fact that every nonempty interval of real numbers contains both rational an! ational numbers to show that the function 1 ix is cational £E)= V6 it is imational is discontinuous at every pon. ‘b) Is f right-continuous or left-continuous at any point? 54, If functions (2) and a(x) are continuous for 0 = x = 1, could {2/2 possibly be discontinvous ata point of (0, 1? Give reasons for your answer 55, Ifthe product function h(x) = f(x) + #(8) scontinuous atx =O, rust (x) and ¢(x) be continuous atx = 07 Give teasons for your “Asersk denotes a challenging problem, ‘86. Give an example of functions f and g, both continuous at x = 0, for which the composite f og is discontinuous at x = 0, Does this contradict Theorem 8 Give reasons for your answer, 57 Is it true that a continuous function that is never zer0 on an interval never changes sign on that interval? Give reasons for ‘58. Is it true that if you stretch a rubber bund by moving one end to Fight and the other tothe left some point of the band will end up in its original position? Give reasons for your answer, 59, A fixed point theorem. Suppose that a function f is continu: fous on the closed interval [0, 1] and that 0 = f(x) <1 for every -x in [0,1], Show that there must exist a number c in (0, 1] such, that fo) =e (c is called a fixed point of f) 60. The sign-preserving property of continuous functions. Let {f be defined on an interval (a, 6) and suppose that f(c) #0 at Some c where f is continuous. Show that there is an interval (©—8, €+8) about ¢ where f has the same sign as /(0) 1.6 Tangent Lines 97 ‘except ¢. That and the condition f(@) #0 are enough to make {different from zero (positive or negative) throughout an entire iterval, 61. Explain how Theorem 6 follows from Theorem 1 in Section 1.2. 62, Explain how Theorem 7 follows from ‘Theorems 2 and 3 in Sec- tion 1.2 % Solving Equations Graphically Use a graphing calculator or computer grapher to solve the equations in Exercises 63-70. 8 —3x-1=0 64, 2x8 28 65. x(¢= I= 1 (oneroo) 66, x! 61. fat VTFR 4 68 x) 15¢+41=0 (dhree roots) 69. cosx =x (one rool). Make sure you are using radian mode. Notice how remarkable this conclusion is. Although {is defined throughout (a,b) itis not required to be continuous at any point 70, 2sin-x =x (three roots), Make sure you are using radian mode, ross er ES TT RAN 1.48 Lis tangent to the cirde at P if it passes through P perpendicular to radius oP. Tangent Lines This section continues the discussion of secants and tangents begun in Section 1.1 We calculate limits of secant slopes to find tangents to curves, What /s a Tangent to a Curve? For circles, tangency is straightforward. A line L is tangent to a circle at a point P if L passes through P perpendicular (o the radius at P (Fig. 1.48), Such a Tine just touches the circle. But what does it mean to say that a line L is tangent to some other curve C at a point P? Generalizing from the geometry of the circle, we might say that it means one of the following. 1. L passes through P perpendicular to the line from P to the center of C: 2. _L passes through only one point of C, namely P. 3. L passes through P and lies on one side of C only. While these statements are valid if C is a circle, none of them work consistently for more general curves. Most curves do not have centers, and a line we may want to call tangent may intersect C at other points of cross C at the point of tangency (Fig. 1.49 on the following page). “To define tangency for general curves, we need a dynamic approach that takes into account the behavior of the secants through P and nearby points Q as Q moves toward P along the curve (Fig, 1.50 on the following page). It goes like this: 1. We start with what we can calculate, namely the slope of the secant PQ. 2. Investigate the limit of the secant slope as Q approaches P along the curve. 3. Ifthe limit exists, take it to be the slope of the curve at P and define the tangent to the curve at P to be the line through P with this slope. This is what we were doing in the fruit fly example in Section 1.1 98 Chapter 1: Limits and Continuity > A “Lmeets Cony t P ‘but snot angen to C ‘Tangent How do you curve? id a tangent toa This was the dominant mathematical question of the early seventeenth century and it is hand {o overestimate how badly the scientists of the day wanted to know the answer. In ‘optics, the tangent determined the angle at ‘which a ray of light entered a curved lens. In ‘mechanics, the tangent determined the Lirection of a body's motion at every point long its path. In geometry, the tangents to two curves at a point of intersection etermined the angle at which the curves intersected. Descartes went so far as 10 say that the problem of finding a tangent to a curve was “the most useful and most general problem not only that F know but even that # hhave any desire to know.” Lis tangent wo Cat P bat ies on two sides of C essing Cat I Listangentto Cx Pot mets € a sever pois. 1.49 Exploding myths about tangent lines. Secants 1.50 The dynamic approach to tangency. The tangent to the curve at P isthe line ‘through P whose slope is the limit of the secant slopes as Q-> P from either side. EXAMPLE 1 Find the slope of the parabola y ‘an equation for the tangent to the parabola at this point. at the point P(2, 4), Write Solution We begin with a secant line through P(2, 4) and Q2+h, (2+ h)?) nearby. We then write an expression for the slope of the secant PQ and investigate ‘what happens to the slope as Q approaches P along the curve: 4 Weds 4—4 Secant slope = 2” ad Btn . ah+4. tah i If h > 0, Q lies above and to the right of P, as in Fig. 151. If h <0, Q lies to the left of P (not shown). In either case, as Q approaches P along the curve, approaches zer0 and the secant slope approaches 4: Him (+4) = 4. We take 4 t0 be the parabola's slope at P 1.51 Diagram for finding the slope of the parabola y = x* at the point 2, 4) (Example 1). 1.52 The tangent slope is lim {00+ 4) ~ flr) ms a How to Find the Tangent to the Curve y = F(x) at (xo, Yo) 1. Calculate f(%0) and (9 + h). 2 Calculate the slope £6504) = Fo) a : 3. Ifthe limit exists, find the tangent line as y = yy + m(x — xo). 1.6 Tangent Lines 99 ‘The tangent to the parabola at P is the line through P with slope 4: Y= 444 Ge — 2) Pointstope equation yaar a Finding a Tangent to the Graph of a Function To find a tangent to an arbitrary curve y = f(x) at a point P(x, f(x») we use the same dynamic procedure. We calculate the slope of the secant through P and a point Q(x +h, f (xo + h)). We then investigate the limit of the slope as h > 0 (Fig. 1.52). 1 the limit exists, we cal it the slope of the curve at P and define the tangent at P to be the line through P having this slope. Definitions ‘The slope of the curve y = f(x) atthe point P(o, f()) is the number Lle0+h) = flea) i m= lim jim, (provided the limit exists) ‘The tangent line to the curve at P is the line through P with this slope. Whenever we make a new definition it is a good idea to try it on familiar objects to be sure it gives the results we want in familiar cases. The next example shows that the new definition of slope agrees with the old definition when we apply it to nonvertical lines. EXAMPLE 2 Testing the definition ‘Show that the line mx +b is its own tangent at any point (xo, mx +6), Solution We let f(x) = mx +b and organize the work into three steps Step 1: Find f(x) and fry +h). S40) = mip +b F(X +H) = m(xo +) +b = mxy + mh +b 100 Chapter 1: Limits and Continuity Pierre de Fermat (1601-1665) ‘The dynamic approach to tangency, invented bby Fermat in 1629, proved to be one of the seventeenth century's major contributions to caleulus. Ferma, a skilled linguist and one of his century's greatest mathematicians, tended to confine his writing to professional correspondence and to papers written for personal friends. He rarely wrote completed 0', the slope ap- proaches oo and the tangent becomes increasingly steep (Fig. 1.54). We see this again as @ — 0”, As a moves away from the origin, the slope approaches (> and the tangent levels off. a Rates of Change The expression Exercises 1.6 101 is called the difference quotient of f at x9, If the difference quotient has a limit as h approaches zero, that limit is called the derivative of f at x9. If we interpret the difference quotient as a secant slope, the derivative gives the slope of the curve and tangent at the point where x =p. If we interpret the difference quotient as an average rate of change, as we did in Section 1.1, the derivative gives the function's rate of change with respect to x at the point x = x. The derivative is one of the ‘wo most important mathematical objects considered in calculus. We will begin a thorough study of it in Chapter 2, EXAMPLE 4 _ Instantaneous speed (Continuation of Section 1.1, Examples 1 and 2) In Examples 1 and 2 in Section 1.1, we studied the speed of a rock falling frely 1.54 The tangent slopes, steep near the from rest near the surface of the earth, We knew that the rock fell y = 160? feet ctigin, become more gradual as the point during the first 1 seconds, and we used a sequence of average rates over increasingly aera short intervals to estimate the rock's speed at the instant r = 1. Exactly what was the rock's speed at this time? All of these refer to the same thing. Solution We let f(t) = 16°, The average speed of the rock over the interval 1. The slope of y= f(a) t= 25 between ¢= 1 and1 = 1+ seconds was rae eaten LOH) = FQ) _ 160-41" = 160? _ 160 +2) _ 16 2, The ate of ehange of 1) with h rpatoraren : 7 ee “The rock's speed atthe instant = 1 was tim Lath = Feo) Him, 16(h +2) = 16(0 4+ 2) = 32 fee. a bs (Our original estimate of 32 fUsee was right. a Exercises 1.6 Slopes and Tangent Lines In Exercises 1-4, use the grid and a straight edge to make a rough estimate of the slope of the curve (in y-units per a-unit) at the points Py and P. Graphs can shift during a press run, so your estimates may be somewhat different from those in the back of the book. RY 4 x 102 Chapter 1: Limits and Continuity In Exercises 5-10, find an equation forthe tngent tothe curve atthe given point. Then sketch the curve and tangent together. =8 1,3) 6 y=G- DHL By hy a2, an a2 By In Exercises 11-18, find the slope ofthe function’s graph atthe given point, Then find an equation for the line tangent to the graph there. f=" 29) 1 fa) =x-28, =D B= 45. G9) 4 ea =5, 2.2) IS A =P, 28) 16 i =P+31, 4) 1. fe)= Vi, 4.2) 18. fx) =VEFT, 6.3) In Exercises 19-22, find the slope of the curve atthe point indicated, =a Tangent Lines with Specified Slopes ‘At what points do the graphs ofthe functions in Exercises 23 and 24 have horizontal tangents? Be f@=rt4e-1 24. a(x) 25, Find equations of al lines having slope —1 that are tangent to ‘the curve y = 1/(x ~ 1). 26, Find an equation of the straight line having slope 1/4 that is tangent tothe curve y = VF. Rates of Change 27. An objec is dropped from the top of 100-m-high tower. Is height aboveground after 1 seconds is 100 — 4.91 m. How fast is falling 2 se after iis dropped? 28. AL seconds afer lift-off, te height of «rocket is 37? How fast is the rocket climbing after 10 sec? 29, What she ate of change ofthe en fice (A = 27) wih respost ot radivs when the radi 310, Whatis the ato change ofthe volume ofa all (V with respect to the rads when the radi is r= 2? ~ ax Testing for Tangents 1. Doss the graph of asin (fx), x #0 sey 0, x=0 hhave a tangent atthe origin? Give reasons for your answer. 32. Does the graph of in, x40 aw) Bay have a tangent at the origin? Give reasons for your answer. Vertical Tangents ‘We say thatthe curve y = (x) has a vertical tangent atthe point where x = 9 if Tim.» (f+) ~ ft0))/h = 00 oF —. Vertical tangent at x = 0: tin LOLM=FO) _ im LOH SO) ‘No vertial tangent at x jim 20+ 20) rs) ih 1 He oes not exist, because the limit is co from the right and —oo from the left. 38, Does the graph of “xed f=} 0 x=0 re) havea vertical tangent at the origin? Give reasons for your answer. 34. Does the graph of , 0, r<0 veo=[t i50 have a vertical tangent atthe point (, 1)? Give easons for your % Grapher Explorations—Vertical Tangents 18) Graph the curves in Exercises 35-44. Where do the graphs appear to have vertical tangents? ') Confiem your findings in (a) with limit calculations 35. as 36. y= xt 37. ya's 38, ye a8 39. yaar 2 40. y h— 5x29 41. P—@- 1's 4 yx $e le CHAPTER Questions to Guide Your Review 103 -vii. x0 a 5 44. y = JE=aT ay yay yeh es © CAS Explorations and Projects Use @ CAS to perform the following steps forthe functions in Exer: cises 45-48, 48) Plot y = f(x) over the imerval xy — 4 x9? 7. What exactly does lim,_.», (8) = L mean? Give an example in ‘which you find a > 0 for a given f,L,, and ¢ > 0 in the Foxmal definition of limi. 8, Give formal definitions of the following statements, fe) a) Tim b) lim» fo) =5 © lima f@) +00 4) tims FO) 9. What conditions must be satisfied by a function if it is to be continuous at an interior point ofits domain? at an endpoint? 10, How ean Tooking atthe graph of a function help you tell where the function is continuous? LL, What does it mesn for a function to be rightcontinuous at a point? left-continuous? How are continuity and one-sided cont: rity related? 12, What can be said about the continuity of polynomials? of rational functions? of trigonometric functions? of rational powers and algebraic combinations of functions? of composites of functions? of absolute values of functions? 13, Under what circumstances ean you extend a function f(x) t0 be continuous a a point x = c? Give an example. 14, What foes it mean fora function tobe continvous on an interval? 18. What does it mean fora funetion 1o be continuous? Give examples to illustrate the fact that a funtion that is not continuous on its ‘entire domain may still be continuous on selected intervals within the domain 104 Chapter 1: Limits and Continuity 16, What property must function f that is continuous on an interval [a,b] have? Show by examples that need not have this propecty i cis discontinuous at some point of the interval 17. Itis often said that @ function is continuous if you ean draw its graph without having to lilt your pen from the paper. Why is that? CHAPTER PRACTICE EXERCISES 18. What does continuity have to do with solving equations? 19, When is a line tangent to a curve C at a point P? 20. What isthe significance of the formula tim LOE = rr Limit Calculations and Continuity 1. Graph the function Ioxs-1 mH -lex<0 faye} i r=0 -z Orel ers = Then discuss, in complete detail, limits, one-sided limits, continu- ity, and one-sided continuity off at each ofthe points x and I. Are any ofthe discontinuities removable? Explain. 1,0, 2, Repeat the instructions of Exercise 1 for 0, ee Vx, Ox <1 soy = [Me O=t 1 rel 43. Suppose that f(s) and g(x) are defined forall and that lic f(2) = —Tand iy ve g(8) = 0.Findthe limits > ofthe following functions a) 34) » ror sa) 9 f@)+20 one w-7 ©) cos(ets) © feo! 4. Suppose that f(s) and g(x) are defined forall x and that lim. f(x) = 1/2 and lim,» g(x) = V2. Find the limits as +0 of the following functions a) =) Dy gtx) fle) 9 f@) +a) & 1/f0) © x+fe) f@- esx In Exercises 5 and 6, find the value that lim,..9g() must have ifthe given limit statements hold sty (258) 6 im, (xm et) In Exercises 7-10, find the limit of g(x) as x approaches the indicated value 7. tim, (4g¢ey)!? = 2 8 lim i Oa ons + e@) 3841 5-8 9. tim ey 10, im, go a Vac (ayes eo 0, yasx 2 (asx 0, asx 1 14. tim = why a n 18 fi, O4"=8 19, On what intervals are the following functions continuous? a) fo) hee x18 b) x8 @ gtx) = key a 20, Can f(x) = (x2 — I/lx* ~ 1) be extended to be continuous at x=1 or =1? Give reasons for your answers. (Graph the funetion—you will find the graph interesting) Grapher Explorations—Continuous Extensions In Exercises 21-24, graph the function to see whether it appears to hhave a continuous extension to the given pont a. It it does, use ‘TRACE and 200M to find a goed candidate forthe extended fane- tion's value at , Ifthe funetion does not appear to have a continuous extension, can it be extended o be continous fom the right or let? 1f so, what do you think the extended functions value shuld be? Scos Bie 2 ne = + HD, 22, 6) 24. key Grapher Explorations—Roots 28, Let fis) = =~ 4) Show that must have a zero between —1 and 2. Db) Solve the uation fx) = 0 graphically with an error of at rost 10°! It can be shown that the exact value ofthe solution in (b) CHAPTER ‘Additional Exercises-Theory, Examples, Applications 105 1, v@\"" 1 ve@y\"” G8) +( ) [Evaluate this exact answer and compare it with the value determined in (b), 26, Let f(x) = 3° — 2 42. 8) Show that f must have a zero between —2 and 0. ) Solve the equation f(x) = 0 graphically with an error of at, rmrost LO‘ ©) It-can be shown that the exact value of the solution in (b) ( z Evaluate this exact answer and compare it with the value determined in ADDITIONAL EXERCISES—THEORY, EXAMPLES, APPLICATIONS 3 3 Aa) lime f(4) =5, must fle) b) IF (0) =5, must lim... FO) Give reasons for your answers. 2 Can lime (fO/gC4)) exist even if ime fe Time g(2) =0? Give reasoos for your answer. 4. Assigning a value to ©, The rules of exponent tll us that ) = 1 if a is any number different from zero, They also tell us that 0" = 0 ifm is any postive number, If we tried to extend these rules to include the case 0°, we would get conficting resus, The fis rale would say while the second would say 0° ‘We ate not dealing witha question of right or wrong hee. Neither rule applies as it stands, so there fs no contradiction, We could, infact, define 0? to have any value we wanted as long 38 ‘we could persuade others to agree. ‘What value would you ike Ot have? Here ace two exam- ples that might help you to decide, (See Exercise 4 for another example) Ba) CALCULATOR Calevlate 2! for x 1,0.01,0.001, and 1s your calculator can go. Write down the value you get each time. What pattern do you see? ‘GRAPHER Graph the function y = x (as y =x) for0 = x = 1. Bven though the function is not defined forx = 0, the ‘raph will approach the y-axis from the right. Toward what y-value does it seem 10 he headed? Zoom in to estimate the value more closely. What do you think itis? Bo) 4. A reason you might want 0? to be something other thar O or 7. AS the number x increases through positive values, the numbers 1/x and 1/(in x) both approach zero, What happens to the umber Says Q)" . 1x increases? Here are wo ways to find out Bay CALCULATOR Evaluate f for x = 10, 100, 1000, and so on, as far as your ealculator can reasonably go. What pattern do you see? GRAPHER Graph f in variety of graphing windows, in- cluding windows that contain the origin. What do you see? Use TRACE to read y-values along the graph. What do you find? Chapter 6 will explain what is going on 5. Lorente contraction. In relativity theory the length ofan object, say a rocket, appears, to an observer, to depend on the speed at ‘hich the object is traveling with respect 10 the observer. If the ‘observer measures the rocket’ length as Ly a rest, then at speed w the rockets length will appear to be the bs Labyf1- 5. eowaney yoa Here, ¢ © 3 x 10" m/sec isthe speed of light a vacuum. What happens to Las v increases? Find lim, ..- L- Why was the left hnand limit needed?

S-ar putea să vă placă și